You are on page 1of 61

CLASS -XII

BOARD COMPANION
PHYSICS
CONTENTS
Chapter Name Page No.
Electrostatic Potential and Capacitance
Theory .................................................................................................................................................................. 1
Exercise-1 .............................................................................................................................................................. 19
Exercise-2 .............................................................................................................................................................. 43
Answer key ........................................................................................................................................................... 47

SOLUTIONS
EXERCISE-2

Electrostatic Potential and Capacitance ................................................................................................. 48


1
Electrostatic Potential
and Capacitance

NCERT Extract

INTRODUCTION
2. Second, in bringing the charge q from R to P, we
We use electric energy for running our DVD player, 
computer, ACs etc. here we will discuss work and apply an external force Fext just enough to counter
energy of the forces generated from interaction of   
the repulsive electric force FE (i.e,Fext  FE). This
electric charges. Along with this we will be discussing
means that there is no net force on or acceleration of
some components like capacitors that use the concept the charge q when it is brought from R to P, i.e., it is
of potential for their application.
brought with infinitesimally slow constant speed.

ELECTROSTATIC POTENTIAL Work Done and Potential energy


Coulomb force between two (stationary) charges, like
Work done by external forces in moving a charge q
the gravitational force, is also a conservative force.
from R to P is
Thus, like the potential energy of a mass in a
gravitational field, we can define electrostatic P
  P
 
potential energy of a charge in an electrostatic field.  
WRP  Fext .dr   FE .dr
R R
Work done by a charge in external electric field Work done by the external force is equal to negative

Consider the field E due to a charge Q placed at the of the work done by the electric force, and gets fully
origin. Now, imagine that we bring a test charge q stored in the form of potential energy of the charge
from a point R to a point P against the repulsive force q.
on it due to the charge Q. This will happen if Q and q  If the external force is removed on reaching P, the
both are positive or both are negative. Let us take Q, electric force will take the charge away from Q.
q>0.  The stored energy (potential energy) at P is used to
provide kinetic energy to the charge q in such a way
that the sum of the kinetic and potential energies is
R
 conserved.
q

P Potential Energy Difference


Q At every point in electric field, a particle with charge
A test charge q(>0) is moved from the point q possesses a certain electrostatic potential energy,
R to the point P against the repulsive force on the work done increases its potential energy by an
it by the charge Q(>0) placed at the origin.
amount equal to potential energy difference between
points R and P. Thus, potential energy difference
Assumptions for the above system
1. First, we assume that the test charge q is so small that U  U P  U R  WRP
it does not disturb the original configuration, namely (Note here this displacement is in an opposite sense
the charge Q at the origin (or else, we keep Q fixed at to the electric force and hence work done by electric
the origin by some unspecified force). field is negative, i.e., -WRP).
Electrostatic Potential and Capacitance

Therefore, we can define electric potential energy The external force at every point of the path is to be
difference between two points as the work required equal and opposite to the electrostatic force on the
to be done by an external force in moving (without test charge at that point.
accelerating) charge q from one point to another for
electric field of any arbitrary charge configuration.
(i) We can see from the Equation U  U P  U R  WRP Note

that the work done by an electrostatic field in moving (i) The above function is a scalar quantity .
a charge from one point to another depends only on
(ii) The unit of electric potential is joule/coulomb,
the initial and the final points and is independent of
which is also known as volt (V). Its dimensional
the path taken to go from one point to the other. This
is the fundamental characteristic of a conservative formula is M1L2T-3A–1.
force. (iii) The electric field at infinite distance due to a
(ii) Also from the same equation we get potential energy finite charge distribution will be zero.
difference in terms of the physically meaningful (iv) One can obtain the function (V) for all points
quantity work. Here, potential energy so defined is in the field, which is defined as the static electric
undetermined to within an additive constant. Thus here potential of the respective points, with respect to
we need to understand that the actual value of potential
the infinite position.
energy is not physically significant; it is only the
difference of potential energy that is significant. We
can always add an arbitrary constant  to potential
Electric Potentials due to Different Types of Charge
energy at every point, since this will not change the
Distributions
potential energy difference:
As already discussed, electrostatic potential at a point
(U P  )  (U R  )  U P  U R can be taken as the amount of work done by an
Now we have a freedom in choosing the point where external force, in bringing a unit positive charge from
potential energy is zero. A convenient choice is to infinity to that point; without any change in its kinetic
have electrostatic potential energy zero at infinity. energy.
With this choice, if we take the point R at infinity, we Let us now discuss the Electric Potential for some
get WP  U P  U   U P . This equation provides us specific charged systems:
Potential due to a point charge
with a definition of potential energy of a charge q at 
any point. Potential at a point with position vector r due to a
Potential Energy of charge in the presence of point charge Q placed at the origin is given by
external charge Field 1 Q
V(r) 
Potential energy of charge q at a point (in the presence 40 r (For relative permittivity =1)
of field due to any charge configuration) is the work
done by the external force (equal and opposite to the Y
electric force) in bringing the charge q from infinity
to that point. A
r
Electrostatic potential (V)
Q
Work done by an external force in bringing a unit X
O
positive charge from infinity to a point = electrostatic
potential (V) at that point. SI unit of potential is volt or joule/coulomb.
In other words, the electrostatic potential (V) at any Note: Potential at a point is arbitrary (within an
point in a region with electrostatic field is the work additive constant), and it is the potential difference
done in bringing a unit positive charge (without between two points, which is physically significant.
acceleration) from infinity to that point.
Conditions Derivation of potential charge due to a point
To obtain the work done per unit test charge, we should charge
take an infinitesimal test charge  q, obtain the work Consider a point charge Q at the origin (as shown in
done  W in bringing it from infinity to the point and figure). For definition, take Q to be positive. We wish
to determine the potential at any point P with position
determine the ratio  W/  q.

2
Electrostatic Potential and Capacitance

vector r from the origin. For that we must calculate


q1 q2
the work done in bringing a unit positive test charge p2
p1 q3
from infinity to the point P. For Q > 0, the work done p3
against the repulsive force on the test charge is
p
positive. Since work done is independent of the path, q0
we choose a convenient path-along the radial direction
Potential at point p depends on charge q1, q2, and q3 and
from infinity to the point P.
also the distance of these the charges from point p.

 Potential due to a uniformly charged spherical


P'
shell
+IC
r' We have studied in the previous chapter that the
r'
electric field outside a charged spherical shell is equal
P to the electric field generated, as if the entire charge
r
of the shell is concentrated at the center of the shell.
O Q
If the radius of the shell is equal to R and the total
Fig. Work done in bringing a unit positive test charge
from infinity to the point P, against the repulsive force charge residing on the shell is equal to q then for a
of charge Q (Q > 0), is the potential at P due to the charge Q
point outside the spherical surface.
1 q
At some intermediate point P’ on the path, the V (r  R)
4 0 r
electrostatic force on a unit positive charge is
The electric field inside the sphere is equal to zero.
Q 1  As a result the electric potential will be uniform for
r' (1)
40 r '2
all points lying inside the sphere and its value will be
where r̂' is the unit vector along OP'. Work done equal to he electric potential on the surface of the
against this force from r' to r' + r' is spherical shell. The value of the electric potential at
the surface of the sphere and inside will be,
Q
W   r ' (2)
4 0 r '2 1 q
V (r  R)
The negative sign appears because for r '  0, W 40 R
is positive. Total work done (W) by the external force Potential Due to an Electric Dipole
is obtained by integrating Eq. (2) from r' =  to r' = ra As studied in the previous chapter an electric dipole
r is a pair of equal and opposite charge q and -q
Q 1
r
Q Q
W   dr '   separated by a distance 2a. Its dipole moment vector

4 0 r ' 2
4 0 r '2 
4 0 r
p has magnitude (2qa) and is in the direction of the
(3) dipole axis from -q to q.
This, by definition is the potential at P due to the
P
charge Q r
2a
Q P
V(r)  (4) -q o +q
40 r
The formula of the electric potential due to an electric
Potential due to more than one charge in space dipole can be given as,

 
 1 p.rˆ
For a charge configuration q1,q2...,qn with position
   V r  (for r  2a)
vectors r1 , r2 ,...., rn , the potential at a point P is given 40 r 2
 
by the superposition principle where r is the unit vector in the direction of OP  r
1 q q q
V  ( 1  2  ...  n ) (for relative perm ittivity  1) Derivation of an expression for Potential due to
4  0 r1P r2 P r nP
an Electric Dipole
  ...
 ) (for relative permittivity 1) Consider that the origin is at the centre of the dipole.
where r1p is the distance between q1 and P, r2p is the It is known that the electric field obeys the
distance between q2 and P and so on. superposition principle. Since potential is related to
3
Electrostatic Potential and Capacitance

the work done by the field, electrostatic potential also The electric potential of a dipole is then given by
follows the superposition principle. Thus, potential 
1 p · rˆ
due to the dipole is the sum of potentials due to the V (r >> a) (7)
4 0 r 2
charges q and –q:
The above Equation (7) is, as indicated,
P approximately true only for distances large compared
r1 to the size of the dipole, so that higher order terms in
q  
a/r are negligible. For a point dipole p at the origin.
r
Eq. (7) is, however, exact.
 Potential on the dipole axis ( = 0, ) is given by
r2
2a  1 p
p O V (8)
40 r 2
(Positive sign for  =0, negative sign for  .) The
–q potential in the equatorial plane () is zero.
Quantities involved in the calculation of
potential due to dipole Note

1 q q 1. If q   and a  0 in p =2qa then the dipole


V    (1)
40  r1 r2  is called a point dipole.
2. The above equation gives us the exact value of
Where r1 and r2 are the distances of the point P from
the electric potential for a point dipole, while
q and – q, respectively.
giving approximate value for a dipole system
By geometry,
other than a point dipole.
r12  r 2  a 2  2ar cos  3. For  = 0 or  i.e. any point along the dipole
r22  r 2  a 2  2ar cos 
(2)
1 p
axis V   4 2
We take r much greater than a (r >>a) and retain terms 0 r
only upto the first order in a/r 4. For a point along the equator of dipole,  =/
2, V = 0
 2a cos  a 2   2a cos  
r12  r 2 1   2   r 2 1   (3)
 r r   r  EQUIPOTENTIAL SURFACE
Similarly, If the electric potential along every point of any
imaginary surface in an electric field is same, then
 2a cos  
r12  r 2 1   such a surface is called an equipotential surface.
 r 
Using the Binomial theorem and retaining terms upto Properties of equipotential surface
the first order in a/r ; we obtain,  No work is done in moving a charge from one point
to another on an equipotential surface.
1/ 2
1 1  2a cos   1 a   The electric field intensity vector is always
 1    1  cos   4
r1 r  r  r r  perpendicular to the equipotential surface passing
through the point.
1/ 2 Equipotential surfaces in some cases
1 1  2a cos   1 a 
 1    1  cos   5 (a) If we consider the point charge to be at the center
r2 r  r  r r 
of the sphere, the surface of the sphere will be an
Using Eqs. (1) and (5) and P = 2qa, we get equipotential surface.
1 q2a cos  p cos  (b) The uniform electric field existing along the X-
V  (6)
40 r2 40 r 2 axis can be represented by electrical field lines drawn
 parallel to the X-axis. The equipotential surfaces will
Now, p cos  = p · rˆ
be along the YZ plane perpendicular to the X-axis.
where r is the unit vector along the position vector.. (c) The equipotential surfaces for a dipole system
 and a system made up of two equal positive charges
OP are shown in figure (c)
4
Electrostatic Potential and Capacitance

– +
V = +30 V
V = +50 V

+
+
+

V = +70 V

(a) (b) (c)

(a) For a point charge, concentric spheres centred at the location of the charge are equipotential surface as shown in the figure
above. The arrows represent the field lines. Note : The voltage have been given arbitrary value (b) Equipotential surface for a
uniform electric field (c) Some equipotential surfaces for field of a dipole

ELECTRIC POTENTIAL AND FIELD STRENGTH ELECTRIC POTENTIAL ENERGY OF A SYSTEM OF


If Vb-Va is the potential of b.w.r.t.a, that is change in CHARGES
potential when a point moves from a to b The electrostatic potential energy of system of point
charges is the work required to assemble this system
Wa b
b
 

of charges by bringing them in from an infinite
Vb  Va    E.dr (1)
q0 distance.
a

(a) Potential Energy of a system of two charge


  b


Vb  Va   E.dr Consider charges q1 and q2 initially at infinity and
(2)
determine the work done by an external agency in
a
bringing the charges to the given locations.
Suppose, first the charges q1 is brought form infinity
b b
  
  dV   E.dr  (3) to the position r1 . There is no external field against
a a which work needs to be done, so work done in
  
bringing q1 from infinity to r1 is zero.
or dV  E.dr (4)
The charge q1 produces a potential in space given
We can also write this as
by:
+ dV = – Ex dx – Ey dy – Ez dz (5)
1 q1
Using partial derivatives this gives us V1 
40 r1P
V V V
Ex   , Ey   , Ez   (6) Where r1P is the distance of a point P in space from
x y z
the location of q1. From the definition of potential,
In terms of unit vector we have
work done in bringing charge q2 from infinity to the
  V  V  V   
E    i j k  (7)
position r2 is q2 times the potential at r2 due to q1.
 x y z 
1 q1q 2
Equations (6) and (7) give the relation between V Work done on q2 =
4 0 r12

and E
Where r12 is the distance between points 1 and 2.
Note
Partial derivative is not a part of the CBSE syllabus Since electrostatic force is conservative, this work
hence the analysis (using calculus) is only useful gets stored in the form of potential energy of the
in cases of uniform field. system. Thus, the potential energy of a system of two
charges q1 and q2 is

5
Electrostatic Potential and Capacitance

Potential Energy of a system of more than two


1 q1q 2
U charges
4 0 r12 If the system is made of n electric charges, then
potential energy is given as :
U = U12 +U13 +...+UIn +U23 +U24 +...+U2n + ...+Un-1Un
q2
n n kq i q j
r1 2
U  i 1
U ij  
i 1 rij
i j i j

q1
Potential Energy in an Uniform Electric field
Fig. Potential energy of a system of charges q1and q 2is Potential Energy of a single charge
directly proportional to the product of charges and
inversely to the distance between them. When a charge q is placed at some point P, with

position vector r relative to some origin, in an
(b) Potential Energy of system of three charges 
external field E (such that q has no significant effect
Let us consider three charges q1 , q2 and q3 located at on this external field) then the potential energy of q
     
r1 , r2 , r3 respectively. To bring q1 first from infinity at is given as; U( r ) = q V ( r ) where V ( r ) is the
 
to r1 , no work is required. Next we bring q2 from external potential at the point r .

 Note: The distribution of V ( r ) depends on the
infinity to r2 . As before, work done in this step is
distribution of external electric field .
 1 q1q 2
q 2 V1 (r2 )  (1)
4  0 r12 Potential Energy of two charges
When charges q1 and q2 are placed at points with
The charges q1 and q2 produce a potential, at any  
position vectors r1 and r2 (relative to some origin)
point P is given by 
respectively, in an external field E (such that the
1  q1 q 2  two charges have no significant effect on this external
V1.2    
4 0  r1P r2P  (2) 
field) then the potential energy of two charges at r1
Work done next in bringing q3 from infinity to the 
and r2 is given as
 
position r3 is q3 times V1, 2 at r3 . 
  q 1 q 2 (where V(r) is the
U  q 1 V(r1 )  q 2 V (r2 ) 
 1  q1q3 q 2 q3  40 r
q3 V1,2 (r3 )     
40  r13 r23  external potential at the point r .)

Note
r23 
q q2 The distribution of V( r ) depends on the distribution
3
of external electric field.
r13 Here the total potential energy is due to the External
r12 
field E and also due to the mutual interaction
q between the two charges.
1

Fig.:A system of three charges q q 2& q 3 located at


1 Potential energy of dipole kept in a uniform
r , r2 , r3 respectively .
1
electric field
Consider a dipole with charges q1=-+q and q2 = –q
The total work done in assembling the charges at the placed in a uniform electric field E , as shown in the
given locations is obtained by adding the work done figure.
in assembling pair of charges individually As seen in the last chapter, in uniform electric field,
the dipole experiences no net force; but experiences

1  q1q 2 q1q 3 q 2 q 3  a torque  given by
U     (3)   
40  r12 r13 r23    pE

6
Electrostatic Potential and Capacitance

(1) When a conductor is placed in an electric field, the


free electrons migrate in a direction opposite to the
 electric field direction because of the force acting on
E
q them. The electrons get deposited on one side of the
x
surface of the conductor and positive electric charge
acos 
 acos gets deposited on the other side of the conductor.
p
–q This rearrangement of electrons generates an electric
2a field inside the conductor. This electric field is in a
direction which is opposite to the original electric
Fig: Potential energy of a dipole field. The rearrangement of the electrons stops once
in a uniform external field the internal electric field equals the external electric
 field. Thus, the electrons get deposited on the surface
which will tend to rotate it (unless p is parallel or
of the conductor, where they remain stationary.

antiparallel to E ). Suppose an external torque ext is
Note
applied in such a manner that it just neutralizes this
torque and rotates it in the plane of paper form angle (i) If we draw a Gaussian surface inside the
0 to angle 1 at an infinitesimal angular speed and conductor, then since the electric field on the
Gaussian surface is zero, the net charge enclosed
without angular acceleration. The amount of work
by such a Gaussian surface is also zero.
done by the external torque will be given by (ii) Stationary electric charge distribution is
1 1
induced on the surface of the conductor.
W   ext ()d   pE sin  d (iii) The electric field at any point inside the
0 0
conductor is zero.
= pE(cos 0 – cos 1) (2) (iv) For every point on the outer surface of the
This work is stored as the potential energy of the conductor, the electric field is perpendicular to the
system. We can then associate potential energy U () surface (this is because there is no electrical force
acting along the tangential direction to any point
with an inclination of the dipole. Similar to other
of this surface so there is no component of electric
potential energies, there is a freedom in choosing the field along the tangential direction.)
angle where the potential energy U is taken to be
zero. A natural choice is to take 0 = / 2. We can Electrostatic shielding
then write, If we presume a hollow conductor placed in an
     external electric field, the electric field is zero in the
U( )  pE  cos  cos     pE cos   p.E
 2  interior portion of the conductor. The electric charges
are residing on the outer surface of the conductor . If
CONDUCTORS AND ELECTRIC FIELDS we consider a spherical Gaussian surface in the
Let us consider a conducting material placed in a hollow region of the conductor, electric field will be
 be equal to zero on the Gaussian surface and also
uniform electric field E .A conductor consists of free inside the Gaussian surface, since it is devoid of any
electrons and positive ions fixed at the lattic point. electrical charge. This phenomenon is called
Gaussian surface E
Electrostatic Shielding.

da r When we are sitting in a car which is struck by


lightning the person sitting in the car is not affected
by lightning, as the interior of the car is hollow and
L this acts like an electrostatic shield.
Placing of a charge on the surface of the conductor
Gaussian surface
If we place an electric charge on the surface of a
conductor then it gets distributed only on the outer
E
surface of the conductor. Let us consider a Gaussian
surface very close to the surface of the conductor
(which is shown by dotted liens in figure (b)).
(a)

7
Electrostatic Potential and Capacitance

Properties in a nutshell
Properties for a conductor placed in an electric field:
+ +
+ ++ 
+
1. Electrostatic field E is zero in the interior of a
+ +++
+ +++ conductor.
+
+ +
+ 
+ + 2. Outside the surface of a charged conductor, E is normal
+
+ +
++ +  
++ + + +
+ + + + ++
+
to the surface given by E  n where n is the unit vector
0
(b)
along the outward normal to the surface and  is the
surface charge density.
Direction of field at the surface of the conductor. 3.` Charges on a conductor can reside only at its surface.
4. Potential is constant within and on the surface of a
Since the electric charges are stationary the direction conductor.
of electric field will be perpendicular to the surface
of the conductors (which is also shown in figure (b) Dielectrics and Polarization
Polar and non-polar molecules

and its magnitude will be  . The molecules of a substance may be polar or non-
0 polar.
In order to understand the above fact, let us consider Non-Polar - In a non-polar molecule, the centres of
a Pill box shaped Gaussian surface on the surface of positive and negative charges coincide. The molecule
the conductor. The charge enclosed by the Gaussian then has no permanent (or intrinsic) dipole moment.
surface =A.  Examples of non-polar molecules are
The total flux passing through this surface EA oxygen (O2) and hydrogen (H2) molecules which,
because of their symmetry, have no dipole moment.
A
 As per Gaussian’s Theorem, EA  Polar Molecule - On the other hand, a polar molecule
0 is one in which the centres of positive and negative
 charges are separated (even when there is no external
 E
0 field). Such molecules have a permanent dipole
moment. An ionic molecule such as HCl or a molecule
If  is not uniform along the surface, then we will of water (H2O) are examples of polar molecules.
have to substitute the corresponding value of  at
respective points in order to calculate E.
O
Non-polar H H C O
Field and charge distribution when charge is
placed in the cavity of conductor H2 CO2
As shown in the figure (c), a positive electric charge
is placed in the cavity (hollow region) of the H O
conductor. This charge will induce charges along the
outer surface of the conductor and on the surface Polar
Cl P H H P
bounding the hollow region in such a way that the
field will be zero in the interior portion of the HCl H2O
conductor. Some example of polar and non-polar molecular

Dielectrics
+ ++
+ + + Dielectrics are non-conducting substances. In contrast
+ +
+ ++
+ + +
+
to conductors, they have no (or negligible number
+ + + + of) free charge carriers.
+ + + + +
+ + Induced Dipole Moment in dielectric
++ E=0 +
++ + + + + In a dielectric, free movement of charges is not
+ + ++ ++
possible. It turns out that the external field induces
(c)
dipole moment by stretching or re-orienting

8
Electrostatic Potential and Capacitance

molecules of the dielectric. The collective effect of Dipole Moment in polar molecules
all the molecular dipole moments is like that of the A dielectric with polar molecules also develops a net
net charges on the surface of the dielectric, which dipole moment in an external field but for a different
produce a field that opposes the external field .Unlike reason. In the absence of any external field, the
in a conductor the opposing field created by a different permanent dipoles are oriented randomly
dielectric, so induced, does not exactly cancel the due to thermal agitation; so the total dipole moment
external field. The net electric field of the system is is zero. When an external field is applied, the
only reduced. The extent of the effect depends on individual dipole moments tend to align with the field.
the nature of the dielectric. When summed over all the molecules, there is a net
dipole moment in the direction of the external field,
i.e., the dielectric is polarised. The extent of
E0 +
free free+ polarisation depends on the relative strength of two
E0 Ein + E0
E0 + Ein= 0 + + mutually opposite factors: the dipole potential energy
Conductor in the external field tending to align the dipoles with
E0
the field and thermal energy tending to disrupt the
+
  + alignment. There may be, in addition, the ‘induced
E0 E in p + E0
E0 + Ein 0 + + dipole moment’ effect as for non-polar molecules,
Dielectric but generally the alignment effect is more important
Difference in behaviour for polar molecules. Thus in either case, whether polar
of a conductor and a dielectric or non-polar, a dielectric develops a net dipole
in an external electric field moment in the presence of an external field.

Dipole Moment in non-polar molecules Polarization


In an external electric field, the positive and negative The dipole moment per unit volume is called
charges of a non-polar molecule are displaced in 
polarisation and is denoted by P .For linear isotropic
opposite directions. The displacement stops when the  
external force on the constituent charges of the dielectrics, P = xe E where xe is a constant
molecule is balanced by the restoring force (due to characteristic of the dielectric and is known as the
internal fields in the molecule). The non-polar electric susceptibility of the dielectric medium.
molecule thus develops an induced dipole moment.
The dielectric is said to be polarized by the external Field inside polarized dielectric
field. Let us consider, for simplicity, a rectangular dielectric

slab placed in a uniform external field E 0 parallel
Linear isotropic dielectrics
to two of its faces. The field causes a uniform
We consider only the simple situation when the 
induced dipole moment is in the direction of the field polarisation P of the dielectric. Thus every volume

and is proportional to the field strength. (Substances element v of the slab has a dipole moment P v in
for which this assumption is true are called linear
isotropic dielectrics). The induced dipole moments the direction of the field. The volume element v is
of different molecules add up giving at net dipole macroscopically small but contains a very large
moment of the dielectric in the presence of the number of molecular dipoles. Anywhere inside the
external field. dielectric, the volume element v has no net charge
(though it has net dipole sits close to the negative
E=0 E=0 E - 0 E– 0 charge of the adjacent dipole. However, at the
surfaces of the dielectric normal to the electric field,
there is evidently a net charge density. As seen in
Figure, the positive ends of the dipoles remain un
(a) Non-Polar Molcculs (b) Polar Molccles neutralised at the right surface and the negative ends
at the left surface. The unbalanced charges are the
A dielectric develops a net dipole in an external
electric field. induced charges due to the external field. Thus the
(a) Non-polar molecules, (b) Polar molecules. polarised dielectric is equivalent to two charged
surfaces with induced surface charge densities, say

9
Electrostatic Potential and Capacitance

p and  p . Clearly, the field produced by these Capacitor


surface charges opposes the external field. The total The arrangement in which two conductors of arbitrary
field in the dielectric is, thereby, reduced from the shape and volume, are arranged close to one another,
case when no dielectric is present. We should note but separated from each other, is called a capacitor.
The conductors are known as the plates of the
that the surface charge density  p arises from bound
capacitor. The conductor which is positively charged
(not free) charges in the dielectric. is called the positive plate and the conductor which
is negatively charged is called the negative plate.
E The S.I. unit of capacitance is coulomb/volt. The
above unit is also called farad, named after the great
+ + + + scientist Michael Faraday. The smaller units of farad
+ + + +
are called microfarad ( F  106 F ) and picofarad
+ + + +
(pF = 10–12 F).
+ + + +
– p p
+ + + +
Parallel Plate Capacitor
+ + + + A parallel Plate Capacitor has to metallic plates
+ + + + having identical area. These plates are kept parallel
+ + + + to each other. Let Q be the electric charge on the
capacitor. The surface charge density will be
P
  Q/A .
A uniformly polarised dielectric
amounts to induced surface Let d be the separation between the two plates.
charge density, but no volume
charge density Field inside a parallel plate capacitor
For a smaller separation between the plates we can
CAPACITANCE AND CAPACITOR consider that the charge distribution is uniform and
Consider an isolated conducting sphere. As we that a uniform electric field is generated between the
plates.
continue placing some positive electric charge on the
The electric field generated between the two plates
sphere as shown in figure it results in increase in the
due to charge +Q (i.e. it will be directed from +Q
electric potential (V) on the surface of the spheres as
towards -Q)

well as increases the electric field E in the vicinity
of the sphere. During this entire process, the ratio of 
the amount of charge (Q) on the sphere and the + 20 –
electric potential (V) of the sphere remains constant.
This ratio is called its capacitance (C). 
20
+ +

Conducting 0
sphere
+ +

Non-Conducting E1 
2 0
stand
The electric field generated in the region between
Note: Capacitance depends only on the shape and the plates by the -Q charge will be
size of the body and on the permittivity of the 
E2 
surrounding medium. It is independent of the charges 2 0
and the electric potential.

10
Electrostatic Potential and Capacitance

The resultant electric field between the two plates Cdielectric C


K 
will be equal to. C vacuum C0

   where Cvacuum =capacity of a capacitor when there is


E  E1  E 2    (Seefigure)
20 20 0 vacuum between the plates
Cdielectric = capacity of the same capacitor when
Q Q a dielectric is placed between the plates
E  (   )
0 A A Thus, the dielectric constant of a substance is the
factor by which the capacitance increases from its
If V is the potential difference (p.d) between the two
vacuum value, when dielectric is inserted fully
plates, then V=Ed
between the plates.
Substituting the value of E from above equation, we
Q Derivation of Capacitance with dielectric
have V   A d
0 Parallel plate capacitor with the dielectric
Consider a parallel-plate capacitor with plate area A
The capacitance of the parallel plate capacitor can
and separation d between the plates (as shown in the
Q 0 A figure). A dielectric slab of dielectric constant K is
be given as C  
V d inserted in the space between the plates. Suppose,
Note: The distance d is kept less in comparison with the slab almost completely fills the space between
the dimension of the plate. By doing so, the non- the plates.
uniform electric field due to the irregular distribution A charge Q is given to the positive plate and –Q to
of the charges near the edges can be minimized. the negative plate of the capacitor. The electric field
polarizes the dielectric so that induced charges +Qp
CAPACITOR WITH DIELECTRIC and -Qp appear on the two faces of the slab.
If the medium between the plates of a capacitor is
filled with an insulating substance (dielectric), the Qp Qp –Q
electric field due to the charged plates induces a net +Q
E0 +
dipole moment in the dielectric. This effect called
Ep +
polarization, gives rise to a field in the opposite
direction. +

+ – + – E +
+ – + – + – d
+ – + –
+ – + – + –
+ – + – The electric field at a point between the plates due to
+ – + – + –
+ – the charges +Q, -Q on the capacitor plates is
+ –
+ – + – + –
–  Q
+ – + E0   ...(1)
+ – + – + – 0 A0
+ – + –
+ – + – + – in a direction left to right in the figure.
+ – + –
– + – + – From the definition of dielectric constant, the
+
resultant field is
(a) (b) E Q
E 0  ...(2)
(a) Electric field lines with vacuum between the plates, K 0 AK
(b) The induced charge on the force of the dielectric reduce
The potential difference between the plates is
the electric field
Qd
Dielectric V = Ed 
0 Ak
Due to this the potential difference between the plates
is reduced. Consequently, the capacitance C increases The capacitance is
from its value C0 when there is no medium (vacuum) Q K0 A
The dielectric constant K is defined by the relation C   KC0 ...(3)
V d
11
Electrostatic Potential and Capacitance

0 A q  t  0 A
where C0  C
q
 q/ d  t   
is the capacitance without the
d V 0 A  K t
dt
dielectric. Thus, K
The capacitance of a capacitor is increased by a factor 0 A
of K when the space between the plates is filled with Or C (5)
d  t(11/ K)
a dielectric of dielectric constant K .
COMBINATIONS OF CAPACITORS
Capacitance in case of dielectric slab of thickness The desired value of capacitance can be obtained by
t (t<d ) combining more than one capacitor, We shall discuss
Consider a parallel plate capacitor having each plate two ways in which more than one capacitor can be
of area A and separated by a distance d. connected.
When there is vacuum between the two plates, the
capacitance of the parallel plate capacitor is given by 1. Series Combination of Capacitors:
There is equal amount of charge Q deposited on each
0 A
C0  (1) capacitor; but the p.d between their plates is different
d
(if the capacitance of each of the capacitors is
Suppose that when the capacitor is connected to a different)
battery, electric field of strength E 0 is produced
between the two plates of the capacitor. Further,
C1 C2 C3 Cn
suppose that when dielectric slab of thickness t (t < + – + – + – + –
Q Q Q Q
d) is introduced between the two plates of the
capacitor (Figure), the electric field reduces to E due V1 V2 V3 Vn
to polarisation of the dielectric.

Dielectric Slab
A B +
+ V –
+
+ (n)
+
+
+ V=V1+V2+V3+.....+Vn
+
t Q Q Q Q Q
   ....  (from C  )
d C1 C2 C3 Cn V
Now, between the two plates of the capacitor; over a V 1 1 1 1
distance t, the strength of electric field is E and over      ...
Q C1 C2 C3 Cn
the remaining distance (d - t), the strength of electric
field is E0. If V is a potential difference between the If C is the resultant capacitance of C1, C2, .....Cn which
V 1
plates of the capacitor, then
are connected in series then  .
V = E t + E0 (d - t) Q C

E0 E0 1 1 1 1 1
Since K or E , we have (2)      ... 
E K C C1 C2 C3 Cn

E0  t  Note: The value of the combination is smaller than


V t  E 0 (d  t)  E 0  d  t   (3) the smallest of the values of the capacitors, which
K  K
are connected in series.
 q
Now, E0  
0 0 A 2. Parallel Combination of Capacitors:
In this arrangement of the capacitors the charged
q  t 
 V d  t   (4) accumulated on each of the capacitors is different,
0 A  K
while the potential difference between them is the
Hence, capacitance of a parallel plate capacitor on same and is equal to p.d. between the common points
introduction of dielectric slab is given by A and B of such a connection.

12
Electrostatic Potential and Capacitance

+ –
Q1 C1 Let q and  be the charge and potential difference
respectively, at an intermediate stage during he
+ – C2 charging process; then  = q/C. At the stage the work
A Q2 B
dW required to transfer an additional element of
+ – charge dq is
Q3 C3
q dq
dW   dq 
V C
The total work W needed to increase the capacitor
(b)
charge q from zero to a final value Q is
Now, Q1 = C1V, Q2 = C2V, Q3 = C3V
W Q
1 Q2
Q   dW 
C 0
The total electric charge, Q = Q1 +Q2+Q3 q dq 
Q = C1V + C2V + C3V 0
2C
Q = (C1+C2+C3)V (work to charge a capacitor)
 Therefore, the total capacitance of the above This is also equal to the total work done by the electric
parallel arrangement, field on the charge when the capacitor discharges.
Q Then q decrease from an initial value Q to zero as
C  C1  C2  C3 the elements of charge dq “fall” through potential
V
differences  that very from V down to zero.
In general,
If we define the potential energy of an uncharged
Q capacitor to be zero, the work (W) in Eq. is equal to
C  C1  C2  C3  ....  Cn
V the potential energy U of the charged capacitor. The
Hence, by connecting capacitors in a parallel final stored charged is Q = CV, so we can express U
combination, a resultant capacitance can be obtained, (which is equal to W) as
whose value is equal to the sum of all capacitances
Q2 1 QV
connected in parallel. U  CV 2 
2C 2 2
ENERGY STORED IN A CHARGED CAPACITOR (potential energy stored in a capacitor)
Work needs to be done to charge a capacitor. This
work done is stored in the form of potential energy Energy Stored in a Capacitor in a terms of Energy
of the charges which is called the energy of the Density of Electric Field
capacitor. The energy stored in a capacitor,
Let the two conducting plates of the capacitor have 1
U CV 2
the charge Q and – Q. The energy stored in this 2
configuration is given by: If A is the area of the plates and if d is the separation
Q2 1 1 between the plates. The volume of the system
U  CV 2  QV comprising the capacitor = Ad
2C 2 2
The energy stored per unit volume (Density of energy)
Derivation of energy stored in a capacitor
in the region between the plates is
Many of the most important applications of capacitors
depends on their ability of story energy. The electric
potential energy stored in a charged capacitor is just U F 1 0 A v 2 1 V V
E   .   0 ( )( )
equal to the amount of work required to charge it- Ad 2 d Ad 2 d d
that is, to separate opposite charges and place them
1 V
on different conductors. When the capacitor is  E   0 E 2 (  E)
discharged, this stored energy is recovered as work 2 d
done by electrical forces.
We can calculate the potential energy U of a charged Note
capacitor by calculating the work W required to
 The above equation suggests that the energy stored
charge it. Suppose that when we are done charging in a capacitor can be found in terms of the energy
the capacitor, the final charge is Q and the final stored in the electrical field between the two plates.
potential difference is V. From Eq. these quantities  Although the above formula has been obtained for
are related by a parallel plate capacitor the result hold true for an
Q electric field due to any charge distribution.
V
C

13
Electrostatic Potential and Capacitance

SOME IMPORTANT DEFINITIONS AND DERIVATIONS


• Potential energy of dipole: The work done in rotating • Capacitor: A capacitor has two conductors separated
the dipole from one position to another is stored in the by dilectric medium such that it can store large amount
form of energy called potential energy of the dipole. of electric charge in small space.

• Electric Potential: Electric potential of a body is a • Principle of a parallel plate capacitor: Capacitance
physical quantity which determines the flow of charge of a charged conductor is increased by bringing another
from that body to another body. uncharged or low potential conductor near it while the
two conductors remain separated with some dielectric
• Electric potential at a point in the electric field: It is medium between them.
defined as the work done per unit charge in moving a
unit positives test charge from infiniy to that point • Principle of van de Graaff generator:
against the electrostatic force of the field irrespective (i) Charge remains on the outer surface of a shperical
of the path followed. shell.
(ii) Pointed surfaces have larger charge densities.
• Electric Potential Difference: between any two points
in an electric field is the negative line integral of electric • Electric potential at a distance r from A point
field intensity between these points along any path. charge.
Let A be the point at a distance rA from charge q.
• Volt: Potential at a point is one volt if one joule of Let W be the work done in moving a test charge q0
work is done in moving one coulomb of charge from from infinity to A, then potential is given by
infinity to that point in the electric field.
W
VA = q
• Principle of superposition of potentials: The net 0
potential at any point in the field of a group of charges Let the test charge at any instant be at P small work
is given by the algebraic sum of their individual 
potentials at that very point. done in moving it through a small distance d l is given
by
  
• Equipotential surface: A surface having same dW = F.d l  (–q E)dl  –q Edl cos180
0 0
potential at every point due to charged distributon is 
called equipotential surface. Since distance r decrases in the direction of d l .
Therefore, dl can be taken as – dr.
• Electric Potential energy: The work done on a charge  dW = – q0 E dr
in bringing it from infinity to a point in an electric field
1 q
against the electrical force is called electrical potential But E = 4  2
energy. 0 r
q A q0E
• Dieletric: The non-conducting material in which
charges are easily produced on the application of rA
electric field is called dielectric e.g. Air, H2 gas, glass, r
mica, paraffin wax transforme oil etc.  1 q
 dW = – q0  4  2  dr
• Polarization: of a dielectric is the process of getting  0 r 
equal and opposite charges on the two opposite faces 1 qq0
of the dielectric on the application of electic field. = – 4  2
dr
0 r
• Dielectric strength: The maximum value of the electric Work done is moving test charge from  to A is given
field intenstiy that can be applied to the dielectic by
material without its electric break down is called the A rA
1 qq0 1  r–1 
dielectric strength of that material. W =  dW   – 4 0 dr  – qq0  
  r 2 4 0  1 
• Electrical Capacitance: The ability of a conductor to
store charges is known as electrical capacitance or 1  1 1  1 qq0
apacity of a conductor. = 4  qq0  r    4  r
0  A  0 A

• Farad : It is the S.I. unit of capacitance . Capacitance 1 q


is should to be 1 farad if 1 coullomb of charge is V A = 4  r
required to raise the potential through 1 volt. 0 A

14
Electrostatic Potential and Capacitance

1 q q  2l cos  
In genral, V = 4  r V=  
0 4 0  r2 – l2 cos2  
q2l cos 

4 0 (r2 – l2 cos2 )
• Electric potential at any point due to an elctric
dipole-thus-electric potential axial line of a dipole
Consider P to be the point of observation at a distance
p cos 
r from the centre (O) of the electric dipole. Let OP make = 4  (r2 – l2 cos2 ) .......(ii)
 0
an angle  with the dipole moment p and r1, r2 be the
( dipole moment, p = q . 2l)
distances of point P from –q charge and +q charge
respectively (in figure). p cos 
V
Potential of P due to –q charge, If r > > l, then equation (ii) becomes 4 0 r2
1 (q) 1 q 
V 1= 4   r = 4   r Since p cos  = p.r,
ˆ where r̂ is unit vector directed
0 1 0 1
along OP.
 
prˆ
P V
4 0 r2 for r > > 1
r1
r D
r2 If point P lies on the axial line of the dipole i.e.  = 0°
l cos
p
A B V
4 0 r2
–q O p +q then equaiton (ii) becomes
l l
l cos

1
C

or V  ( cos 0° = 1)
Potential of P due to +q charge, r2
• Electric potential at equatorial line of a dipole.
1 q Using above result if point P lies on the equatorial line
V2 = 4   r
0 2 of dipole i.e.  = 90°,
 Potential at P due to the dipole, then using equation (ii), we get
V=0 ( cos 90° = 0)
V = V1 + V2
Potential due to a dipole is zero at a all points on the
(Principle of superposition)
equatorial line of the dipole.
1 q 1 q • Total work done in rotating a dipole in electric field
or V = – 4  r + 4   r or potential energy of a dipole in an electric field.
0 1 0 2
Let the electric dipole having moment p be placed at
an angle  with the direction of electric field E, then
q 1 1 
or V=    torque acting on dipole is given by
4 0  r1 r2 
P
Now draw a perpendicular from A which meets the line
OP at C when produced backward. Also draw BD  on B qE
+q
OP> 2a
Then r1 = AP  CP = OP + OC = r + l cos 
( from AOC, OC = l cos ) qE –q A C
and r2 = BP  DP = OP – OD = r – l cos 
( from BOD, OD = l cos )  = p E sin 
E
Substituting the values of r1 and r2 in equation (i), we if dipoole is rotated throug small angle d then small
get work done is given by
dW =  d = – pE sin d
q  1 1 
V=  (r  l cos )  (r  l cos )  Work done in rotating the dipole to an angle  from the
4 0   initial position say perpendicular to the dirction of th
electric field is given by,
q  r  l cos  – r  l cos   
     
= 4 
0  (r2  l2 cos ) 
W =  pEsin  d = p E   cos   /2  p E  cos  cos  
 2 
 /2
15
Electrostatic Potential and Capacitance

q = q1 + q2 + q3
i.e. W  pE But q1 = C1V, q2 = C2V, q3 = C3V
Note : The initial position of dipole in this expression.  q = C1V + C2V + C3V
= (C1 + C2 + C3) V ...(i)
• Equivalent capacitance of capacitors in series. If C is equivalent capacitance of parallel combination
Let V1, V2 and V3 be the potential differnece across then q = CV
three capacitors C1, C2 and C3 respectively. Hence, eqn. (i) becomes
Then total voltage V = V1 + V2 + V3 CV = (C1 + C2 + C3) V
C1 C2 C3 or C = C1 + C2 + C3
If n capacitors are connected in parallel, the equivalent
capacitance of the combination is

V1 V2 V3 C  C1  C2  C3  .......  Cn
V
q q
Now C1 = V or V1 = C • Capacitance of a parallel plate capacitor.
1 1 Consider two paralel plates of conducting material
separated by distance d.
q q
Similarly, V2 = C and V3 = C because same charge If a voltage V is applied to the capactior, an electric
2 3 field E will set up i.e.
is induced in all the capacitors w here q is the charge
V
given to first capacitor. E= or V = Ed.
d
q q q 1 1 1 
V = C  C  C  q C  C  C   q
1 2 3  1 2 3 But E =  where  is charge density given by,
0 A
q
But V= where C is equivalent capacitance of
C d qd
the combination  V =  = A and
0 0

q  1 1 1 
 =q=     Capacitance of a capacitor
C  C1 C2 C3  is given by
1 1 1 1 q q
  
or C C1 C2 C3 C= = (qd / A  ) q+ + q–
V 0

If n capacitors are connected in series then equivalent + –
 A  A
capacitance of the combination is given by = 0 C= 0 + –
d d + –
1 1 1 1 1
    ............  . If a dielectric medium of
+ –
Ceq C1 C2 C3 Cn dielec tric constant K is
+ –
kept in between the
• Equivalent capacitance of capactiors in parallel. plates then + –
Let three capacitors of capacitances C1, C2 and C3 be + –
 KA
connected in parallel as shown in the figure. C 0 + –
All capacitors will have some potential difference d
across them.
Capactiros C1, C2 and C3 will have differnet amount of • Capacitanced with dielectric slab between the
charges q 1 , q 2 and q 3 respectively. In parallel plates.
combination, total charge q is the sum of the charges Capacitance of a parallel plate capacitor with air as the
stored by each capcitor. medium between two plates of area A lying at a distance
i.e. C
d apart is given by,
1
–q
0 A
+q +
+


+ –
+ –

C=
A
+q +
+
+
C2



–q
B
d
+ –

C3
Let a dielectric slab of thickness t be put between the
+q
+
+
+
+



– –q
plates such that t < d. Due to polarisation, electric field
will reduce from E0 to E.
Potential difference across the capacitor is given by
V V = E0 (d – t) + Et

16
Electrostatic Potential and Capacitance

P Q capacitor, then
+ –
q
+ –
=
+ – A
+ –
q(d  t)
+ – Hence eqn. (ii) becomes, V = 0 A ..(4)
+ –
+ –
q
+ – We know, C=
– – V
+ t –
d q 0 A 0 A
 C= or C =
E0 E q(d  t) (d  t)
Dielectric constant, K = or E = 0
E K Dividing (iv) by (i), we get
E0t C d  t
 V = E0 (d – t) +   1 / 1  
K C0 (d  t)  d 
 q
Now E=  = A C0
0 0 or C= ...(5)
 t
q  1 1  d 
 
 V = A  d  t  
0  K • Energy stored in a capacitor.
q Let a capacitance C be charged to potential V. If q is
 Capactiance of the arrangement = C = i.e. the charge on the plate then
V
q q
0 A C=
V
or V =
C
C
d  t(1  (1 / K)) Small amount of work done by battery in charging the
• Capacitance with condu cting slab between plates. capacitor to small charge dq at constant voltage V is
Capacity of a parallel plate capacitor having vacuum given by
between the plates is given by
q
 A dW = V dq =   dq
C0 = 0 C
d
....(1) q q q
 q 1 1  q2 
W =    dq   dq   
where A = Area of each plate

d = distance of separation between the plates. 0 
C C0 C  2 
0
Let a conducting slab of thicknesss ‘t’ be introduced
between the plates of the capacitor. Equal and opposite
1 q2 1 (CV)2 1
charges appear on the two faces of the conducting slab. or W=   CV2
 2 C 2 C 2
Electric field (E) inside the conducting slab (i.e.
This work done is stored inside the capacitor as
conductor) is zero. potential energy given by,
Now the electric field (E0) exists only in a region of
thickness (d – t). 1
U CV2
 Potential difference between the plates of the 2
capacitor.
• Energy density of a capacitor.
V = E0 (d – t)
1
 Energy stored in a capacitor, U = CV2
But E0 =  2
0
where  is the surface charge density q 1 q2
But C= U=
(d  t) V 2 C
 V= 0 ....(2)
0 A
Again C=
d
If q is the magnitude of the charge on each plate of the and q =  A = 0 EA

17
Electrostatic Potential and Capacitance

Electrostatic potential energy of two capacitors after


  
 E = or  0 E sharing
 0  1 1
U= C1V2  C2V2
2 2
1 (0 EA)2 1
 U=   E2Ad (After sharing both capacitors attain some potential
2 0 A / d 2 0 difference V)
But Ad = Volume of the capactior 1
 Energy stored in a capacitor per unit volume = (C  C2 )V2
2 1
of the capacitor
C1V1 + C2V2
1
i.e. enery density =  E2 But V= C1  C2
2 0
2
• Energy change n two charged capacitors connected 1  C1V1 + C2 V2  1 (C1V1 + C2 V2 )2
in parallel. U = 2 (C1  C2 )  C1  C2
 
2 (C1  C2 )
 
Consider two capacitors having capacitance C1, C2
charges q1, q2 and potentials V1, V2 respectively.
 Step 3
Respective charge on the capacitors is given by
Law of energy
q1 = C1V1 and q2 = C2V2.
On sharing of charges some energy is dissipated in the
Therefore, total charge on capacitors (before sharing)
form of heat.
= q1 + q2 = C1V1 + C2V2.
 Step 1 1
U – U’ = C V2  C2V22 –
Common Potential 2 1 1
Let these capacitors be connected in parallel by thin
metallic wire. (C1V1 + C2 V2 )2
Now the charge will flow from a capacitor having higher 2(C1  C2 )
potential to the capacitor at lower potential. This flow
continues till potential of both the caacitors becomes 1
equal. This equal potential is called common potential = 2(C  C ) [(C1V12 + C2V22)(C1 + C1) –
1 2
(V).
(C1V12 + C2V22)]
If q1’ and q2’ be the charges on capacitors C1 and C2
respectively, after the redistribution of charges, i.e. 1
=
sharing takes place, then 2(C1  C2 )
q1’ = C1V and q2’ = C2V
q1 ' C1 [C12 V12  C1C22 V12  C1C22 V22  C22 V22
 =
q2 ' C2  C12 V2  C22 V22  2C1C2 V1 V2 ]
Now, total charge on the capacitors after connecting
them together is given by C1C2 (V12  V22  2V1V2 )
U – U’ =
q = q1’ + q2’ = C1V + C2V = (C1 + C2)V. 2(C1  C2 )
According to the law of conservation of charges,
Total charge after sharing = Total charge before sharing
C1C2 (V1  V2 )2
(C1 + C2)V = C1V1 + C2V2 or U  U' 
2(C1  C2 )
C1V1 + C2V2
or V= C1  C2

 Step 2
Potential energies
Electrostatic potential energy of two capacitors before
sharing
1 1
U = U1 + U2 = C V2  C V 2
2 1 1 2 2 2

18
Electrostatic potential and capacitance

EXERCISE 1
Previous Years Examination Questions Thus, inner sphere has net potential higher than
Electric potential and Potential difference potential of outer sphere for every value of q and Q.
Five Marks Questions Therefore, when they are connected by a wire, positive
Q.1 A small sphere of radius a carrying a positive charge q charge always flow from inner sphere (at higher
is palced concentrically inside a large hollow potential) to outer sphere (at lower potential)
conducting shell of radius b (b > a). This outer shell
irrespective of the magnitude of charge.
has charge Q on it. Show that if these spheres are
connected by a conducting wire, charge will always
Equipotential Surface
flow from the inner sphere to the outer sphere
irrespective of the magnitude of the two charges. Five Marks Questions
[All India 2009] Q.2 The electric potential as a function of distance 'x' is
shown in the figure. Draw a grph of the electric field E
Sol. Let small sphere has charge q and radius a is placed
as a function of x. [All India 2019]
inside a outer shell of charge + Q and radius b.
V
Insulated
suspension

q a
+Q
b x
0 1 2 3
Electric potential on the small sphere due to its own
dV
charge q. Sol. Electric field E = – .....(i)
dx
1 q For x = 0 to 1, V = kx
V1  . ......(i)
4 0 a x = 1 to 2, V = k
where, q = charge on small sphere x = 2 to 3, V = – kx
a - radius of small sphere
Similarly, electric potential on outer sphere due to its
own charge +E

1 Q
V2  . .....(ii) 1 3
40 b 2 x
–E
where, Q = charge of outer shell
b = radius of outer shell.
Also, same potential exists at every point inside outer whyere k is some constant
shell due to its own charge, + Q. So, using (i) the variation of electric field is shown in
Now, net electric potential at inner sphere of radius a. figure.
Vl =Electric potential due to charge on both sphere.
Q.3 Is the electrostatic potential necessarily zero at a point
1 q 1 Q
Vi = 4  a  4  b ......(iii) where the electric field is zero ? Give an example to
0 0 support your answer. [All India 2019]
Net electric potential at outer sphere due to charge on dV
Sol. The electric field E =
the both spheres. dr
+ + q
+ +
E=0
+ + +

1 q 1 Q
+

V0  .  . ......(iv)
4 0 a 4 0 b
+ +

V = constant
+

q 1 1
+

 Vi – V0 = 4  a  b  ......(v)
++ + +
0  
So, even for a constant electric potential electric field
From Eqs. (iii) and (iv), we get can be zero.
1 1 For example, for a hollow shell, the field inside is zero,
 a < b and >  Vi – V0 > 0 wehreas potential is non zero and constant.
a b
19
Electrostatic potential and capacitance

Potential energy in an External Field Q.5 (a) Three point charges q, –4q and 2q are placed at the
Three Marks Questions vertices of an equilateral triangle ABC of side 'l' as
Q.4 Four point charges Q, q, Q and q are placed at the shown in the figure. Obtain the expression for the mag-
corners of a square of side 'a' as shown in the figure. nitude of the resultant electric force acting on the
FInd the charge q.

Q q A
q

–4q 2q
q a Q B l C

(a) resultant electric force on a chare Q, and (b) Find out the amount of the work done to separate
(b) potnetial energy of this system. [All India 2018] the charges at infinite distance. [All India 2018]
Sol. (a) Force on charge Q ude ot charge q.
1 q(4q)
1 qQ Sol. (a) FAB 
Eq = 4   2 40 l 2
0 a
Force on charge Q ude to another charge Q, 1 4q 2
=
40 l 2
1 Q2
FQ = 
4 0 (a 2 ) 2

1 Q2 1 q(2q) 1 2q 2
FAC = 4 =
=
4 0 2a 2 0 l2 40 l 2

Q q
FAC
a 2
a 120º
q
A
Q
q a Fq
FAB

FQ 60º 60º
Fq –4q 2q
B l C
Net force on charge Q is
 
Angle between forces FAB  FAC is 120º.
Fnet  FQ  Fq2  Fq2  FQ  Fq 2
Magnitude of resultant force,

1 Q2 1 qQ F= 2
FAB  FAC
2
 2FAB FAC cos120º
  2  2
4 0 2a 4 0 a 2
1  q2   1 
 (4)  (2)  2  4  2   
2 2
Q Q  =  2
  2 q  along diagonal 40  2 
4 0a 2  2  l 
(b) Potential energy of the given system, (b) Required work done = Change in potential energy
U = UqQ + UQq + UqQ + UQq + Uqq + UQQ of the system = Uf – Ui
= 4UqQ + Uqq + UQQ = 0 – (UAB = UBC + UCA)
1
4qQ q2 Q2 = [q(4q)  ( 4q)(2q)  (q)(2q)]
   40l
40a 4 0 ( 2a) 4 0 ( 2a)

1 10q 2
1  q Q  2 2 = [4q  8q  2q ] =
2 2 2
  4qQ    40 l 40l
40a  2a 2a 
20
Electrostatic potential and capacitance

Capacitors and capacitance (ii) Let Q be the charge stored in the capacitor
One Mark Questions Q=CV =2×10–6 × 120
Q.6 The given graph shows the variation of charge q versus Q = 24 × 10–5 C
potential difference V for two capacitors C1 and C2.
Both the capacitors have same plate separation but Q.8 Show that the capacitance of a spherical conductor is
plate area of C2 is greater than that C1. Which line (A or 40 times the radius of the spherical conductor.
[Delhi 2010]
B) corresponds to C1 and why? [All India 2014C]
Sol. As we know, to determine the electric field at any point
A at distance r from centre if we apply Gauss's theorem
q q
E=
B 40 ·r 2


q
Sol. q = Cv V Hence, V  E·dr 
40 ·r
q A
 where, 0 = 8.854 × 10–12 C2 N–1 m–2
v d The capacitance of the spherical conductor situated in
q vacuum is given by
C= = Slope = tan 
v
q q
C=   C = 40r.
0 A V 1 q
 Slope ·
d 40 r
A  Slope, d = constant
Hence, the capacitance of an isolated spherical
area of C2 is greater so line A line represent for C2
line B correspond to C1 because slope (q versus V) of B conductor situated in vacuum is 40 times of its radius.
is less than slope of A.
Parallel plate capacitor
Three Marks Questions
Two Marks Questions
Q.7 A capacitor of unknown capacitance is connected
Q.9 What is the area of the plates of 2 F parallel plate
across a battery of V volt. The charge stored in it is 360
capacitors having separation between the plates is
c. When potential across the capacitor is reduced by
0.5 cm? [Al India 2011]
120 V, the charge stored in it becomes 120 C.
Sol. According to the question, separation between the
Calculate [Delhi 2013]
platers = d = 0.5 cm = 0.5 × 10–2 m and capacitance
(i) the potential V and the unknown capacitance C.
C = 2F
(ii) what will be the charge stored in the capacitor, if the
d = 0.5 cm = 0.5 × 10–2 m
voltage applied had increased by 120 V?
0 = 8.854 × 10–12
Sol. (i) We have intial voltage, V1 = V volt and charge stored,
Q1 = 360C 0 A
 C=
Q1 = CV1 ........(i) d
Charged potential, V2 = V – 120
Q2 = 120C Cd 2  0.5  10 2
A=  = = 1.13 × 109 m2
Q2 = CV2 0 8.854 10 12
By dividing Eq. (ii) form Eq. (i), we get

Q1 CV1 360 V Three Marks Questions


  
Q 2 CV2 120 V  120 Q.10 (i) Derive the expression for the capacitances of a
parallel plate capacitor having plate area A and plate
 V = 180Volt separation d. [Delhi 2014]
Q1 360  106 (ii) Two charged spherical conductors of radii R1 and
 C=  = 2 × 10–6 F = 2F R2 when connected by a conducting plate respectively.
V1 180
Find the ratio of their surface charge densities in terms
Hence, the potential, V = 180 Volt and unknown of their radii
capacitance is 2F.
21
Electrostatic potential and capacitance

Sol. (i) Parallel plate capacitor consists of two thin


C
q

q 0 A
conducting plates each of area A held parallel to each or C =
V qd / 0 A d
other at a suitable distance d, One of the plates is
insulated and other is earthed. There is a vacuum where, 0 = 8.85 × 10–12 C2 N–1m–2
between the plates. (ii) Surface charge density is given by
q
=
Area A
4R 2
Surface
charge X After connecting both the conductors, their potentials
density will become equal.
1
+ + + + + + + + + + V1 = V2
E
Kq1 Kq 2
d
 
R1 R2
– – – – – – – – –
2
Surface 1 q Kq
charge Y Earthed [ for spherical conductors V = or V = ]
density 40 R R

2
q R  q1 / 4R12 q1  R 2  R
Suppose, the plate X is given a charge of +q coulomb.  1  1 1= 2 =q  R 
 2
q2 R 2  2 q 2 / 4R 2 2  1  R1
By induction, –q colomb of charge is produced on the
inner surface of the plate Y and q coulomb on the outer
surface. Since, the plate Y in connected to the earth, Q.11 In a parallel plate capacitor with air between the plates,
the +q charge on the outer surface flows to the earth. each plate has an area of 6 × 10–3 m2 and the separation
Thus, the plates X and Y have equal and opposite between the plate is 3 mm. [Foreign 2014]
charges. (i) Calculate the capacitance of the capacitor.
Suppose, the surface density fo charge on each plate (ii) If this capacitor is connected to 100 V supply, what
would be the charge on each plate?
is . We know that the intensity of electric feild at a
(iii) How would charge on the plates be affected if a
point between two plane parallel sheets of equal and
3 mm thick mica sheet of K = 6 is inserted between the
oppoiste charges is /0, where 0 is the permittivity of
plates while the voltage supply remains connected ?
free space. The intensity of electric field between the
Sol. Given, area of each plate, A = 6 × 10–3m2
 Distance between plates d = 3mm = 3 × 10–3 m
plates will be given by, E =  (i) Capacitance of parallel plate capacitor is given by
0

The charge on each plate is q and the area of each plate 0 A


C=
is A. Thus, d
0 = 8.85 × 10–12 C2 N–1 m–2
q q
= and E = ......(i)
A 0 A 8.85 10 12  6  103
C=
Now, let the potential difference between the two plates 3 103
be V volt. Then, the electric feild between the plates is C = 1.77 × 10–11 F
given by (ii) Charge on paralle plate capacitor is given by
Q = CV
V
E or V = Ed Given, V = 100V
d
Now, Q = 17.7 × 10–12 × 100
Subsituting the value of E from Eq. (i), we get Q = 1.77 × 10–9 C .....(i)
qd (iii) Given, K = 6
V= Now, C' = KC
0 A
 Q' = KQ[From Eq. (i)]
 Capacitance of the capacitor is Q' = 6 × 1.77 × 10–9
Q' = 10.62 × 10–9 C

22
Electrostatic potential and capacitance

Q.12 A parallel plate capacitor is charged to a potential Sol. (i) According to question
difference V by a DC source. The capacitor is then
disconnected from the source. If the distance between + –
the plates is doubled, state with reason, how the + –
following will change? [Delhi 2010] + –
E
(i) Electric field between the plates + –
P Q
+ –
(ii) Capacitance
+ –
(iii) Energy stored in the capacitor.
+ –
Sol. After disconnection from battery and doubling the A
d
A
separation between two plates
(i) Chage on capacitor reamins same (a) Electric field due to a plate of positive charge at
i.e. CV= C'V'

C point P = 2
 CV =   V'  V' = 2V 0
2
 Eelctric field between the plates 
Electric field due to other plate = 2
V 2V
0
E' =  Since, they have same direction, so
d ' 2d
  
E' =
V
=E E = 2  2  
d 0 0 0

 Electric field between the two plates remain same. Outside the plate, electric field be zero because of
(ii) Capacitance reduces to half of original value as opposite direction.
(b) Potential difference between the plates is given by
1 C
C  C' =
d 2 d  
V = Ed =   E  
(iii) Energy stored in the capacitor before disconnection 0   0 
form battery
(c) Capacitance of the capacitor is given by ( Q = CV)
2
q
U1 = Q A  0 A
2C C=  
V d d
Now, energy stored in the capacitor after disconnection 0
from battery
(ii) According to question,
q2 q2 2q2
U2    = 2U1
2(C')  C  2C
2   R 2R
2
Energy stored in capacitor gets equal to its initial value.

Five Marks Questions


kq
Q.13 (i) If two similar large plats, each of areaAhaving surface Potential at the surface of radius V=
R
charge densities + and – are separated by a distance
d in air, find the expression for [All India 2016] [ q =  × 4R2]
(a) field at point between the two plates and on outer k4R 2
side of the plats. Specify the direction of the  = k4R = 4kR
R
field in each case.
(b) the potential difference between the plates. Potential at the surface fo radius 2R.
(c) the capacitance of the capacitor so formed. kq
= [ q =  × 4(2R)2 = 16R2]
(ii) Two metallic spheres of radii R and 2R are charged, 2R
so that both of these have same surface charge density
. If they are connected to each other with a conducting k16R 2
So,  8kR
wire, in which direction will the charge flow and why? 2R

23
Electrostatic potential and capacitance

Since, the potential of bigger sphere is more. So, charge Two Marks Questions
will flow from sphere of radius 2R to sphere of radius R. Q.18 1F capacitance connected to a battery of 6 V. Initially
switch S is closed. After sometime S is left open and
Q.14 When a parallel plate capacitor is connected across a dielectric slabs of dielectric constant
dc battery, explain briefly how the capacitor gets K = 3 are inserted to fill completely the space between
charged. [All India 2019] the plates of the two capacitors. How will the (i) charge
Sol. Consider a parallel plate capacitor is connected across and (ii) potential difference between the plates of the
a battery as shown in figure. capacitors be affected after the slabs are inserted?
[Delhi 2011]

– – – –– S
C –
V
+ + +++ +
C2
6V C1 1F 1F

Then the electric current will flow through the circuit.


As the charges reach the plate, the insulating gap does Sol. According to the diagram are when the switch S is
not allow the charges to move further ; hence, positive closed, the two capacitors C1 and C2 in parallel will be
charges get deposited on oen side of the plate and charged by the same potential difference V.
negative charges get deposited on the other side of
the plate. As the voltage begins to develop, the electric S

charges begins to resist the deposition of further


charges. Thus the current flowing through the circuit 6V C1 =1m F C2 =1m F
gradually becomes less and then zero till the voltage of
the capacitor is exactly equal but opposite to the volt-
age of the baterry. This is how capacitor gets charged. So, charge on capacitor C1
q1 = C1V = 1 × 6 = 6C
Effect of Dielectric on capacitance
and charge on capacitor C2
One Mark Questions
q2 = C2V = 1 × 6 = 6C
Q.15 Distinguish between a dielectric and a conductor.
Hence total q = q1 + q2 = 6 + 6 = 12 when switch S is
[Delhi 2012]
opened and dielectric is introduced. Then,
Sol. Dielectrics are non-conductors and do not have free
electrons at all. While conductor has free electrons S
which make it able to pass the electricity through it.
6V C'1 A C'2 B
Q.16 Define the dielectric constant of a medium. What is its
unit? [Delhi 2011C]
Sol. Dielectric
The ratio of applied external electric field and resultant Capacity of both the capacitors becomes K times
electric field is known as dielectric constant K of i.e. C1'  C'2 = KC = 3 × 1 = 3C (as C1'  C'2 )
E0 Capacitor A remains connected to battery
dielectric medium, i.e., K  E  E and it is a
0 P
 V1'  V  6V
dimensionless quantity.
q1'  Kq1  3  6C  18C
Q.17 A metal plate is introduced between the plates of a Capacitor B becomes isolated
charged parallel plate capacitor. What is its effect on
the capacitance of the capacitor? [Foreign 2009]  q '2  q 2 or C'2 V2' = C2V2 or (KC) V2'  CV
Sol. If a metal plate is introduced between the plate of a
charged parallel plate capacitor, then capacitance of V 6
or V2 =   = = 2V
parallel plate capacitor will become infinite. K 3

24
Electrostatic potential and capacitance

Q.19 Figure shows two identical capacitors C1 and C2, each Non-polar molecules have symmetrical shapes e.g. any
of 2F capacitance, connected to a battery of 5 V. non-conducting material.
Initially, switch S is closed. After sometimes, S is left
open and dielectric slabs of dielectric constant K = 5 Three Marks Questions
are inserted to fill completely the space between the Q.21 (i) How is the electric field due to a charged parallel
plates of the two capacitors. How will the (i) charge plate capacitor affected when a dielectric slab is inserted
and (ii) potential difference between the plates of the between the plates fully occupying the intervening
capacitors be affected after the slabs are inserted ? region?
[All India 2011; Delhi 2011] (ii) A slab of material of dielectric constant K has the
same area as the plate of a parallel plates capacitor but
S 1
has thickness d, where d is the separation between
C1 2F C2 2F 2
5V
the plates. Find the expression for the capacitance when
the slab is inserted between the plates.[Foreign 2010]
Sol. When a dielectric medium of dielectric constant K is Sol. (i) The capacitance of capacitor increases to K times of
introduced, original values.
(i) in an isolated (not connected with battery) capacitor,  CV= C'V'
then total charge on capacitor remains same. V
(ii) in a capacitor connected with battery, then potential CV = (KC)V'  V' =
K
difference across the capacitor remains same as that of
 New electric field,
potential difference across battery.
Two identical capacitors C1 and C get fully charged V'  V / K   V  1 E
E' = =  =  =
with 5 V battery initially. d  d  d K K
So, the charge and potential difference on both  On introduction of dielectric medium, new electric
capacitors becomes. 1
q = CV = 2 × 10–6 × 5V = 10 C field E' becomes
K
times of its original value.
and potential difference V = 5 Volt
(ii)  Capacitance of a parallel plate capacitor partially
On introduction of dielectric medium of K = 5 filled with dielectric medium is given by
For C1 (continue to be connected with battery) potnetial 0 A
difference of C1, (V') = 5V C=
(d  t  t / K)
Capacitance of C1'  kc = 5 × 2F = 10F Where, t is the thickness of dielectric medium.

Charge, q' = C'V' = (10 F) (5 V) = 50 C Here, t=


d
For C2 (disconnected with battery) 2
Charge, q' = q = 10C
0 A 0 A
Potential difference, C= 
d d d 1
d  1
V 5 2 2K 2  K 
V' =  =1V
K 5
20 AK
 C = (K  1) d
Q.20 Distinguish between polar and non-polar dielectric.
[All India 2010 C]
Sol. Polar dielectrics Five Marks Questions
A polar molecule which has permanent electric dipole Q.22 (i) Explain using suitable diagrams, the difference in
moment (p) in absence of electric field also polar the behaviour of a
molecules are randomly oriented. e.g. Water, alcohol, (a) conductor and
HCl, NH3 (b) dielectric in the presence of external electric field.
Non-polar dielectrics Define the terms polarisation of a dielectric and write
A non-polar moelcule having zero dipole moment in its its relation with susceptibility.
(ii) A thin metallic spherical shell of radius R carries a
normal state.
25
Electrostatic potential and capacitance

Q (b) Electric flux through the shell.


charge Q on its surface. A point charge is placed at
2
1
its centre C and an other charge + 2Q is placed outside = × magnitude of the charge enclosed by the
0
the shell at a distance x from the centre as shown in the
figure. shell.
Q A 1 Q Q
2Q  =  
C 0 2 20
Q
2
Find Q.23 (i) Obtain the expression for the potential due to an
(a) the force on the charge at the centre of shell and at electric dipole of dipole moment P at a point P on the
the point A, axial line.
(b) the electric flux through the shell. [All India 2015] (ii) Two identical capacitors of plate dimensions  × b
Sol. (i) When a conductor is placed in an external electric and plate separation d have dielectric slabs filled in
field, the free charges present inside the conductor between the space of the plates as’ shown in the figure.
redistribute themselves in such a manner that the
electric field due to induced charges opposes the 
external field within the conductor. This happens until
a static situation is achieved, i.e. when the two fields
cancels each other and the net electrostatic field in the
K
conductor becomes zero. 
In contrast to conductors, dielectrics are non-
conducting substances, i.e. they have no charge
K K
carriers. Thus, in a dielectric, free movement of charges
is not possible. The external field induces dipole /2 /2
moment by stretching molecules of the dielectric. The
collective effect of all the molecular dipole moments is Obtain the relation between dielectric constants K, K1
the net charge on the surface of the dielectric which and K2 is if after Inserting dielectric find Capacitance of
produces a field that opposes the external field. both remains same. [All India 2013]
However, the opposing field is so induced that does Sol. (i) Attempt yourself.
not exactly cancel the external field. It only reduces it. (ii) Let A  area of each plate and C1 and C2 are
The extent of the effect depends on the nature of capacitance of each slab.
dielectric. Both polar and non-polar dielectric develop
0 A
net dipole moment in the presence of an external field. Let initially, C1 = C = = C2
d
The dipole moment per unit volume is called
polarisation and is denoted by P for linear isotropic After inserting respective dielectric slabs
dielectrics. C2' = KC

P = E 0 (A / 2) 0 (A / 2)
and C2' = K1 + K2
d d
where  is susceptibility
(ii) (a) At point C, inside the shell. The electric field 0 A
= (K1 + k2);
inside a spherical shell is zero. Thus , the force 2d
experienced by charge at the centre C will also be zero.
C
 FC = qE (Einside the shell = 0) C2' = (K + K2)
2 1
 FC = 0 From Eqs. (i) and (ii), we get
 3Q  C 1
 1 2  C1' = C2'  KC = (K1 + K2)  K = (K1 + K2)
2 2
At point A, |FA| = 2Q  40 x 2 
 
  Q.24 (i) Aparallel plate capacitor is charged by a battery to a
2 potential. The battery is disconnected and a dielectric
3Q
F= , away from shell. slab is inserted to completely fill the space between the
40 x 2
plates. How will
26
Electrostatic potential and capacitance

(a) its capacitance


1 3 2 4 60
(b) electric field between the plates and    Ceq  F
(c) energy stored in the capacitor be affected? Justify Ceq 60 9
your answer giving necessary mathematical expressions Charge appeared on equivalent capacitor
for each case.
20
(ii) (a) Draw the electric field liens due to a conducting Q  Ceq V  106  90
sphere. 3
(b) Draw the electric field liens due to a dipole.
 Q  600C
[All India 2011]
Sol. Attempt yourself.
Ceq
– –
+ ––
+

– –
– –
–q 90V
– –
– – Conducting sphere Charge on each capaciitor is same as they are in series.
having negative Now, potential drop across C2
charge
Q 600  106
(b) Electric field lines due to an electric dipole are shown V2    20V
C2 30 106
in the figure.
Hence, work done stored as electric potentiaol energy

1
U of capacitor is U  C2 V22

2
+ –q
+q

1
 30 106   20   6 10 3 J
2
U
2

Q.26 A slab of material of dielectric constant K has the same


Combination of capacitors
area as that of the plates of a parallel plate capacitor,
Two Marks Questions
but has the thickness d/2, where d is the separation
Q.25 Calculate the potential difference and the energy stored
between the plates. Find out the expression for its
in the capacitor C2 in the circuit shown in the figure.
capacitances when the slab is inserted between the
Given potential at A is 90 V, C1 = 20 F,
plates of the capacitor. [Delhi 2013]
C2 = 30 F and C3 = 15F.
Sol. Initially, when there is a vacuum between two plates,
[All India 2015]
A 0 A
B then capacitance of the plate is C0  , where, A is
C1 C2 C3 d
the area of parallel plates. Suppose that the capacitor is
Sol. For a series combination of three capacitors C1.C2 and
connected to a battery, an electric field E0 is produced.
C3 the equivalent capacitance Ceq will be
Now, if we insert the dielectric slab of thickness t = d/2,
1 1 1 1 1 1 1 1 the electric field reduces to E.
      
Ceq C1 C 2 C3 Ceq 20 30 15 Now, the gap between plates is divided in two parts,
for distance t, there is electric field E and for the
C1 =20 m F C2 =30 m F remaining distance (d – t) the electric field is E0. If V be
the potential difference between the plates of the
C3 =15 m F capacitor, then V = Et + E0 (d – t)

Ed E 0d d  d
V    E  E 0  t 
90V 2 2 2  2 

27
Electrostatic potential and capacitance

Q.28 A network of four capacitors, each of capacitance 15F,


d  E0 
 V  2  K  E0  is connected across a battery of 100 V as shown in the
 
figure. Find the (i) net capacitance and (ii) the charge
on the capacitor C4: [All India 2012C]
dE0  E 
  K  1  As  K
2K  E 0  C2

 q d q
Now, E 0     A  V  2K .  A  K  1 C1 C3
0 0 0

q 2K 0 A C4
We know that, C  V  d  K  1
100V

Q.27 Determine the potential difference across the plates of Sol. (i) According to the diagram given in the question
the capacitor C1 of the network shown in the figure.
(Assume, E1 > E2) [All India 2013] C2

C1 C1 C3
–q +q C4

100 V
E1 E2
Here C1, C2 and C3 are in series, hence their equilvalent
+q –q capacitance C' is given by
C2
1 1 1 1
  
C' C1 C2 C3 [C1 = C2 = C3 = 15 F]
C1
– + Redrawing the circuit as shown below
q
1 1 1 1
E1 E2   
C' 15 15 15
Sol.
C2
+ – 15
q C'  F
3
Potential difference,  C' = 5 F

q q q q
 E1   E 2  0 or   E1  E 2 C'
C1 C2 C1 C2

 1 1 
q    E1  E 2
C4
 C1 C2 
100 V
 C  C2 
q 1   E1  E 2
 C1C2  Since, C' and C4 are in paralle,
 C = C' + C4 = 5 + 15 = 20
(E1  E 2 ) C1C2
q (ii) Since, C' and C4 are in parallel potential difference
C1  C2 across both of them is 100 V.
q  Charge across C4 is Q4 = C4 × V = C4 × 100C
V1 
C1 = 15 × 10–6 × 100
= 15 × 10–4 C
(E1  E 2 )C2
V1  Q4 = 1.5mC
C1  C2

28
Electrostatic potential and capacitance

Q.29 Two identical parallel plate (air) capacitors C1 and C2


have capacitance C each. The space between their d/2 d/2
plates is now filled with dielectrics as shown in the
C1 C2
figure. If the two capacitors still have equal capacitance,
then obtain the relation between dielectric constants
K , K1 and K2. [Foreign 2011]
C1 C2 K1 K2

I/2 K1
d d
I K

K2 Sol. After introduction of dielectric medium of dielectric


constants K1 and K2, capacitor acts as if it consists of
two capacitors, each having plates of area A and
d d
Sol. The capacity of condenser is proportional to the area d
separation connected in series combination for
and inversely proportional to the distance between its 2
plates. If a medium of dielectric constant K is filled in 0 A
C1 = .....(i)
the space between the plates, its capacity beocmes K d
times the capacity when there is air between the plates.
1 1 1
After inserting the dielectric medium, let their  
C3  K10 A   K 2 0 A 
capacitances become C1' and C 2' .  d/2   d/2 
   
For C1

C1' = KC ......(i) 1 1  1 1 
   
C3  0 A   2K1 2K 2 
For C2  d 
 
K10 (A / 2) K 2 0 (A / 2)
C2'  
d d 1 1  K 2  K1 
  
C2 acts as if two capactiors each of area A/2 and C2 2C1  K1 K 2 
separation d are connected in parallel combination.
 2K1 K 2 
 AK K  C2 = C1  K  K 
C2'  0  1 2  1 2 
d  2 2 
The capacitors will be in series.
 K  K2   0 A 
C2'  C  1  .......(ii)  C  d  Q.31 Figure shows a sheet of aluminium foil of negligible
 2   
thickness place between the plates of a capacitor. How
According to the problem, will its capacitance be affected, if
(i) the foil is electrically insulated?
C1'  C2'
(ii) the foil is connected to the upper plate with a
conducting wire? [Foreign 2011]
 K1  K 2  K1  K 2
KC = C    K=
 2  2

Q.30 You are given an air filled parallel plate capacitor C1.
The space between its plates is now filled with slabs of
dielectric constant K1 and K2 as shown in figure. Find
the capacitance of the capacitor C2 if area of the plates
is A and distance between the plates is d. Sol. (i) The system will be equivalent to two identical
[Foreign 2011] capacitors connected in series combination in which
two plates of each capacitor have separation half of
29
Electrostatic potential and capacitance

the original separation. Thus, new capacitance of each  (C1 – C2)2 = 25 × 10–10 – 4 × 4.5 × 10–10
capacitor = 7 × 10–10

 1  (C1 – C2) = 7  1010 = 2.64 × 10–5


C' = 2C  C  d 
   C1 – C2 = 2.64 × 10–5
 C and C' are in series On solving Eqs. (i) and (ii), we get
C1 = 35 F and C2 = 15 F
2C  2C Q1 = C1V = 35 × 10–6 × 100 = 35 × 10–4 C
 Cnet = =C
2C  2C Q2 = C2V = 15 × 10–6 × 100 = 35 × 10–4 C
Cnet = C (Original capacitor)
(ii) System reduces to a capacitor whose separation Q.33 A network of four capacitors each of 12 F capacitance,
reduces to half of riginal one. if connected to a 500 V supply as shown in the figure.
 New capacitance, C' = 2C
C2

Three Marks Questions C1 C3


Q.32 Two capacitors of unknown capacitances C1 and C2 are
connected first in series and then in parallel across C4
battery of 100 V. If the energy stored in the two 500 V
combinations is 0.45 J and 0.25 J respectively, then
Determine
determine the value of C1 and C2. Also, calculate the
(i) the equivalent capacitance of the network and
charge on each capacitor in parallel combination.
(ii) the charge on each capacitor.
[All India 2015]
[HOTS; All India 2010]
Sol. When the capacitors are connected in parallel, Sol. In a series combination, where there is no division of
equivalent capacitance, CP = C1 + C2. charge,
The energy stored in the combination of the capacitors,
1 1 1 1
  
1 Cs C1 C2 C3
EP = CPV2
2
In a parallel combiantion, where potential difference is
1 same. Cp = C1 + C2 + C3 ........
 EP = (C + C2) (100)2 = 0.25 J (i) Here, C1, C2 and C3 are in series, therfore, their
2 1
equivalent capacitance
 C1 + C2 = 5 × 10–9
When the capacitors are connected in series, equivalent 1 1 1 1
  
capacitance, C' C1 C2 C3

C1C2 C 12
CS = C' = = = 4F
C1  C2 3 3
The energy stored in the combination of the capacitors, Now, C' and C are in parallel combiantion.
 Cnet = C' + C
1 = 4F + 12F = 16F
ES = C V2
2 S Cnet = 16F
(ii) Being C' and C are in parallel, 500 V poential
1 C1C2
 ES = 2 (C  C ) (100)  0.45 J
2
difference is applied across them,
1 2  Charge on C'
q1 = C'V
1 C1C 2
  (100)2 = 0.45 J = (4 F) × 500 = 2000 C
2 C1  C2  C1, C2 and C3 capacitors each will 2000 C charge.
 C1C2 = 0.045 × 10–4 × 5 × 10–5 × 2 Charge on C4, q2 = C × V
= 4.5 × 10–19 = 12 × 500
(C1 – C1)2 = (C1 + C2)2 – 4C1C2 = 6000 C

30
Electrostatic potential and capacitance

Q.34 Four capacitors of values 6F,6F, 6F and 2F are


(20 F)  C
connected to a 6 V battery as shown in the figure. (4 F) =
20  C
6 F 6 F (20 + C) = 5C 4C = 20
 C = 5 F
(ii) Charge on capacitor (equivalent)
6 F
q = (4 F) × 12 = 48 C
2 F
same charge 48 C lies on both the capacitors.
(iii) Potential drop across 20 F capacitor

6V q 48 C
V1 =   2.4V
C1 20 C
Determine the
Potential drop across 5 F capacitor
(i) equivalent capacitance of the network.
(ii) charge on each capacitor. [Delhi 2010C] q 48 C
Sol. In series combiantion, charge on each capacitor is the V2 = C  5 C  9.6 V
2
same.
(i) All capacitors of 6 F are in series combination,
Energy in capacitors
then equivalent capacitance is given by
Two Marks Questions
1 1 1 1
   Q.36 A parallel plate capacitor of capacitance C is charged
C' C1 C2 C3 to a potential V. It is then connected to another
uncharged capacitor having the same capacitance. Find
C 6 F
or C' =   2F out the ratio of the energy stored in the combined
n 3
system to that stored initially in the single capacitor.
C' and 2F capacitors are in parallel combination.
[All India 2014]
 Equivalent capacitance
Sol. Let q be the charge on the charged capacitor.
Ceq = C' + 2F
= 2F + 2F
q2
Ceq = 4 F  Energy stored in it is given by U 
2C
(ii) Since, C' and 2F are in parallel combiantion,
therefore, same potential difference 6 V is applied on When another uncharged similar capacitor is
them. connected, then the net capacitance of the system is
 Charge on C' givne by C’ = 2C
q' = C'V' = (2F) × 6 V = 12 C The charge on the system remains constant. So, the
The charge across the each capacitor of 6 F capacitor energy stored in the system is given by
is same and equal to charge across the combiantion i.e.
12 C q2 q2
U'  
Charge on 2F capacitor q = CV = (2 F) (6 V) 2C' 4C
= 12 C
[ C'  2C]
Q.35 The equivalent capacitance of the combination Thus, the required ratio is given by
between points A and B in the given figure is 4F.
U ' q 2 / 4C 1
A B  
U q 2 / 2C 2
20F C
(i) Calculate the capacitance of the capacitor C.
(ii) Calculate the charge on each capacitor if a 12 V Q.37 Two parallel plate capacitors of capacitances C1 and C2
battery is connected across terminals A and B. such that C1 = 2C2 are connected across a battery of V
(iii) What will be the potential drop across each volt as shown in the figure. Initially, the key (k) is kept
capacitor? [Delhi 2009] closed to fully charge the capacitors. The key is now
Sol. According to the question,
thrown open and a dielectric slab of dielectric constant
Capacitors of 20 F and C are connected in series.
K is inserted in the two capacitors to completely fill the
(i) The equivlent capacitance
gap between the plates. Find the ratio of
31
Electrostatic potential and capacitance

(i) the net capacitance and


k
(ii) the energies stored in the combination before and
after the introduction of the dielectric slab.
k [Delhi 2014C] V
C1 C2

V C2
C1 C2 Sol. (i) Given, C1  ...(i)
2

C2 3C
Sol. (i) Given, C1 = 2C2 ...(i) Hence, Cinitial  C1  C2   C2  2 ...(ii)
2 2
Net capacitance before filling the gap with dielectric
slab is given by Net capacitance after filling the gap with dielectric K,

Cinitial  C1  C2 [From Eq. (i)] 3KC2


K, Cfinal 
2
Cinitial  2C2  C2  3C2 ...(ii)
Ratio of net capacitance is given by
Net capacitance after filling the gap with dielectric slab
of electric constant K Cinitial 3C 2 1
  [from Eqs. (ii)]
Cfinal  KC1  KC2  K  C1  C2  [From Eq. (ii)]
Cfinal 3KC 2 K

Cfinal  3KC2 ....(iii)


(i) Given, C1 = 2C2 ...(i)
Ratio of net capacitance is given by Net capacitance before filling the gap with dielectric
slab is given by
Cinitial 3C2 1
  [From Eq. (ii) and (iii)] Cinitial  C1  C2
Cfinal 3KC2 K
[From Eq. (i)]
(ii) Energy stored in the combination before introduction Cinitial  2C2  C2  3C2
of dielectric slab ...(ii)
Net capacitance after filling the gap with dielectric slab
Q2
Uinitial  ...(iv)
of electric constant K
Cfinal  KC1  KC2  K  C1  C2  [From Eq. (ii)]
3C2

Energy stored in the combination after introduction of Cfinal  3KC2


dielectric slab. ....(iii)
Q2 Ratio of net capacitance is given by
Ufinal  ...(v)
3KC2 Cinitial 3C2 1
Ratio of energies stored   [From Eq. (ii) and (iii)]
Cfinal 3KC2 K
U initial K (ii) Energy stored in the combination before introduction

U final 1 [From Eqs. (iv) and (v)] of dielectric slab

Q2
Uinitial 
Q.38 Two parallel plate capacitors of capacitances C1 and C2 3C2
such that C1 = C2/2 are connected across a battery of V ...(iv)
volts as shown in the figure. Initially, the key (k) is kept Energy stored in the combination after introduction of
dielectric slab.
closed to fully charge the capacitors. The key is now
thrown open and a dielectric slab of dielectric constant Q2
Ufinal 
K is inserted in the two capacitors to completely fill the 3KC2
gap between the plates. Find the ratio of ...(v)
(i) the net capacitance and [Delhi 2014C] Ratio of energies stored
(ii) the energies stored in the combination dielectric U initial K
slab.  [From Eqs. (iv) and (v)]
U final 1
32
Electrostatic potential and capacitance

Q.39 Find the charge on the capacitor as shown in the circuit. This work done is stored as electrical potential U of the
6 F [Foreign 2014] capacitor,
Q
Q2

q
UW .dq 
C 2C
0
10 20

Q2 1 1
U  .CV 2  QV [ Q = CV]
2V 2C 2 2

Sol. Total current through the circuit is given by


I = V/R Q.41 Deduce the expression for the electrostatic energy.
Stored in a capacitor of capacitance C and having
Here, V  2volt  R  (10  20)  30
charge Q.
2 1 How will the
 I  A (i) energy stored and
30 15
(ii) the electric field inside the capacitor be affected
2
Voltage across 10 resistor  I 10   10 /15  V when it is completely filled with a dielectric material of
3 dielectric constant K ? [All India 2012]
Charge on the capacitor is given by Sol. Attempt yourself.

 
Q  CV  6 10 6  2 / 3  4C
(i) Energy stored will be decreased or energy stored
1
Q.40 A parallel plat capacitor, each of plate area A and will become times the initial energy..
K
separation d between the two plats, is charged with
charges + Q and –Q on the two plates. Deduce the (ii) Electric field would decrease or
expression for the energy stored in capacitor. E
[Foreign 2013] E' =
K
Sol. The work done in charging the capacitor is stored as
its electrical portential energy. Hence considering a
Q.42 Net capacitance of three identical capacitors in series
capacitor of capacitance C, initially whose two plates
is 1F. What will be their net capacitance, if connected
are uncharged, let q and –q are charges on the two
in parallel?
plates at some Instance during charging, potantial
Find the ratio of energy stored in these two
diffrence at that instant is given by
configurations, if they are both connected to the same
q source. [All India 2011]
V ....(i)
C Sol. If n identical capacitors, each of capacitance C are
If a charge dq is transported in steps from negative connected in series combination give equivalent
charged plates till charges rises to + Q and – Q, then C
work done, capacitance, Cs = and when connected in parallel
n
dW = dq. V ...(ii)
combination, then equivalent capacitance, Cp = nC
From Eqs. (i) and (ii), wen get
Also, form same voltage, energy stored in the capacitor
q is given by
dW  dq    W  dW
C  1
U= CV2 [for constant]
+Q 2
E UC
Cs = 1 F [ n = 3]
dq C
In series combination, Cs =
1 n
2

33
Electrostatic potential and capacitance

In parallel combiantion, Cp = nC As switch S is opened, the potential difference across


According to the problem, capacitor A remains same (V volts).
C = nCs = 3 × 1F = 3F Let potential difference across capactior B be V'. When
For each capacitor, dielectric is introduced with switch S open (i.e. battery
In parallel combiantion. disconnected), the charges on capacitor B remains
CP = nC = 3 × 3 = 9 F unchanged, so
Cp = 9 F QB = CV = C'V'
For same voltage, U  C.
C V
 V' = V= volt
U s Cs Us C / n 1 C' K
 U  C  U  nC  n 2
p p p Initial energy of both capacitors

Us 1 1
1 1 CV2 + CV2 = CV2
  2   Us  1 Ui =
2 2
U p (3) 9 Up 9
Final energy of both capacitors
or Us : Up = 1 : 9
2
1 1 1 1 V
Uf = C'V2 + C'V'2 = (KC) V2 + (KC)  
Three Marks Questions 2 2 2 2 K
Q.43 Two identical parallel plate capacitors A and B are
connected to a battery of V volts with the switch S is 1  1
closed. The switch is now opened and the free space = CV2  K  
2  K
between the plates of the capacitors is filled with a
dielectric of dielectric constant K. Find the ratio of the
1  K2  1 
total electrostatic energy stored in both capacitors = CV  K 
2

before and after the introduction of the dielectric. 2  


[All India 2017]
Ui CV 2 2K
S   2
Uf 1  K 1  K 1
2
CV 2  
2  K 
A B

Q.44 A 12 pF capacitor is connected to a 50 V battery. How


Sol. The given figure is shown below. much electrostatic energy is stored in the capacitor? If
another capacitor of 6 pF is connected in series with it
S with the same battery connected across the
combination, find the charge stored and potential
difference across each capacitor. [Delhi 2017]
V A B
1
Sol. Energy stored in capacitor = C V2
2 1

When switch S is closed. the potential difference across 1


= × 12 × 10–12 × (50)2 J
capacitors A and B are same 2

QA QB = 6 × 25 × 10–10 J = 15 × 10–9 J
i.e. V =  With other capacitor 6 pF in series.
C C
Initial charges on capacitors C1  C2 6  12
Total capacitance (C) = = pF
QA = QB = CV C1  C2 6  12
When the dielectric is introduced, the new capacitance
of either capacitor 12  6
= = 4pF
C' = KC 18

34
Electrostatic potential and capacitance

Charge stored in each capacitor is same and is given


q 60 q 60
by So, Vx = C  5 = 12V and Vy =   3V
Q= CV = 4× 10–12 × 50V = 2 × 10–10C x C y 20
Each of the capacitors will have chrge equal to Q (iii) Electrostatic energy stored in capacitance X
= 2 × 10–10C
1
Potential on capacitors with capacitance 12 pF is Ex = CVx2 .......(i)
2
Q 2  1010
= = V = 16.67V 1
C1 12  10 12 Similarly for Y, Ey = 4CVy2 .......(ii)
2
Potential on capacitor with capacitance 6 pF is From Eqs. (i) and (ii), we get x
2  1010
 V  33.33 V 1
6  1012 CVx2
Ex 2 V2 12  12
Ratio =   x2   4 :1
E y 1 4CV 2 4Vy 4  3  3
y
Q.45 Two parallel plate capacitors X and Y have the same 2
area of plates and same separation between them, X
Q.46 In the following arrangement of capacitors, the energy
has air between the plates while Y contains a dielectric stored in the 6F capacitor is E.
medium of r = 4. Find the value of the following
(i) Calculate the capacitance of each capacitor, if (i) Energy stored in 12F capacitor
equivalent capacitance of the combination if 4F. (ii) energy stored in 3F capacitor
(ii) Calculate the potential difference between the plates (iii) Total energy drawn from the battery
of X and Y. [Foreign 2016]
– +
(iii) Estimate the ratio of electrostatic energy stored in
X and Y. [Delhi 2016]

X Y 6F

3F
12F
+ –
15 V Sol. (i) As given in the question, energy of the 6 capacitor
is E. Let V be the potential difference along the capacitor
Sol. According to question, let the capacitance of X be C, of capacitance 6 F. From the mathematical formula,
so capacitance of Y = r C = 4C [ r = 4]
1
C  4C Since, CV2 = E
(i) Equivalent capacitance = 2
C  4C
(X and Y are in series) 1 E
× 6 × 10–6 × V2 = E  V2 = × 106 .....(i)
2 3
4C 2 4C 4C Since, potential is same for parallel connection, the
=  and it is given that = 4F
5C 5 5 potential thorugh 12F capacitor is also V. Hence,
So, 4C = 20F = capacitance of Y energy of 12F capacitor is

20 1
Capacitance of X = C = = 5F E12 = × 12 × 10–6 × V2 [From Eq. (i)]
4 2
(ii) Charge flowing through the capacitor is given by E
1
= × 12 × 10–6 × × 106 = 2E
4C 45 2 3
q = CV = × 15 = × 15 = 60C
5 5 (ii) Since, charge reamins constant in series, the charge
Now, let the potential difference between plates of on 6F and 12F capacitors combined will be equal to
capacitors X and Y are Vx and Vy, respectively. the charge on 3F capacitor. Using the formula, Q = CV,
we can write
35
Electrostatic potential and capacitance

 (6 + 12) × 10–6 × V = 3 × 10–6 × V' Let us assume that initially both the plates are
V'=6V uncharged. Now, we have to repeatedly remove small
Using Eq. (i) and squaring both sides, we get positive charges from one plate and transfer them to
V'2 = 36V2  V'2 = 12E × 106 other plate.
Now, when an additional small charge (dq) is
1
 E3 = × 3 × 10–6 × 12E × 106 = 18E transferred from one plate to another, the small work
2
done is given by
(iii) Total energy drawn from battery is
Etotal = E + E12 + E3 q'
dW  V 'dq  dq '
= E + 2E + 18E = 21E C
[let chare on plate, when dq charge is transferred is q']
Q.47 Find the ratio of the potential differences that must be The total work done in transferring chare Q is given by
applied across the parallel and series combination of
Q Q
 
q' 1
two capacitors C1 and C2 with their capacitances in the W= dq '  q ' dq '
ratio 1 : 2, so that the energy stored in these two cases 0 C C 0

becomes the same. [All India 2016]


Q
Sol. Total energy stored in series or parallel combiantion of 1  (q ')2  Q2
capacitors is equal to the sum of energies stored in    
C  2  2C
0
individual capacitors. In parallel combination energy
stored in the capacitor This work is stored as electronstic potential energy U
1 1 in the capacitor.
 C1V12  C2 V12 .....(i)
2 2
Q 2 (CV) 2
In series combination energy stored in the capacitor U=  [ Q = CV]
2C 2C
1 C1C2 2
= 2 (C  C ) v2 .....(ii) 1
1 2 U= CV2
2
According to the question, energy in btoh the cases is
The energy stored per unit volume of space in a
same so,
capacitor is called energy density.
1 1  2 C1C2
 2 C1  2 C 2  V1  2(C  C ) V2
2
  1
1 2 CV 2
2 1 0 AV 2
u= u
Ad 2 d2A
V12 C1C2  2 V C1C2
   1 
V22 2(C1  C2 )(C1  C2 ) V2 C1  C2
1
Energy density, u =  E2
C1 1 2 0
But, C  2  C2 = 2C1 Total energy stored in series combination or parallel
2
combination of capacitors is equal to the sum of energies
V1 C1  2C1 2C1 2 stored in individual capacitor.
So,   
V2 C1  2C1 3C1 3 i.e. U = U1 + U2 + U3 +............
(ii) Due to conservative nature of electric force, the
work done in moving a charge in a close path in a
Q.48 (i) Obtain the expression for the energy stored per unit
uniform electric field is zero.
volume in a charged parallel plate capacitor.
(ii) The electric field inside a parallel plate capacitor is
Q.49 Acapacitor of 200 pF is charged by a 300 V battery. The
E. Find the amount of work done in moving a charge q
battery is then disconnected and the charge capacitor
over a closed rectangular loop abcda.
is connected to another uncharged capacitor of 100pF.
[Delhi 2014]
Calculated the difference between the final energy
Sol. (i) The energy of a charged capacitor is measured by
stored in the combined system and the initial energy
the total work done in charging the capacitor to a given
stored in the single capacitor. [Foreign 2012]
potential.

36
Electrostatic potential and capacitance

Sol. Given, C = 200 pF = 200 × 10–12 F and V = 300 Volt Sol. On introduction of dielectric slab in an isolated charged
The energy (initially) stored by the capacitor is capacitor.
(i) The capactiance (C') becomes K times of original
1 1 capacitor as
Ui = CV2 = × 200 × 10–12 × 300 × 300
2 2
0 A K 0 A
= 9 × 10–6 J C= and C' =
d d
The charge on the capacitor when charge through 300
(ii)  Charge remains conserved in this phenomenon.
V battery is
 CV= C'V'
Q = CV
= 200 × 10–12 × 300 CV CV
V' = = [refer part (i)]
= 6 × 10–8C = 60 × 10–9C = 60nC C KC
When two capacitors are connected, they have their V
positive plates at the same potential and negative plates  V' =
K
also at the same potnetial. Let V be the common
potnetial difference. By charge conservation, charge 1
Potential difference decreases and become times
would distribute but total charge would remain K
constant. of original value.
Thus, Q = q + q' (iii) Energy stored initially,
q q q2
 U
C C 2C
q q' Energy stored later,

200 100
q2
q = 2q'  U'  [ C' = KC]
2(KC)
Thus, Q = 2q' + q' = 3q'
where, K = dielectric constant of medium
Q 60nC
So, q' = = = 20 nC
3 3 1  q2  1
 U' =    U' = (U)
and q = 2q' = 40 nC K  2C  K
Thus, final energy
1
q 2 q 2  U' = ×U
Uf =  K
2C 2C '
The energy stored in the capacitor decreases and
1 (40 109 ) 2 1 (20  109 ) 2
=    1
2 200 1012 2 100  10 12 becomes
K
times of original energy..

= 4 × 10–6 + 2 × 10–6 = 6 × 10–6 J


Difference in energy Q.51 Parallel plate capacitor, each with plate area A and
separation d is charged to a potential difference V. The
= final energy – initial energy
battery used to charge it remains connected. A dielectric
= Uf – Ui
slab of thickness d and dielectric constant K is now
= 6 × 10–6 – 9 × 10–6
placed between the plates. What change if any will
= – 3 × 10–6 take place in
Thus, difference in energy is –3 × 10–6 J (i) charge on plates?
(ii) electric field intensity between the plates?
Q.50 A parallel plate capacitor is charged by a battery. After (iii) capacitance of the capacitor?
sometime, the battery is disconnected and a dielectric Justify your answer in each case. [Delhi 2010]
slab with its thickness equal to the plate separation is Sol. On introduction of dielectric slab to fill the gap between
inserted between the plates. How will plates of capacitor completely when capacitor is
(i) the capacitances of the capacitor, connected with battery.
(ii) potential difference between the plates and (i) The potential difference V between capacitors is
(iii) the energy stored in the capacitors be affected? same due to connectivity with battery and hence, charge
Justify your answer in each case. [Delhi 2010] q' becomes K times of original charge as
37
Electrostatic potential and capacitance

q' = C'V' = (KC) (V) = K(CV) = Kq


q' = Kq E V
(ii) Electric field intensity continue to be the same as
potnetial difference and separation between two plates
reamin unaffected as

V
E= O r O r
d
(iii) The capacitance of capacitor becomes K times of
original capacitor. Q.54 A parallel plate capacitor is charged by a battery. After
sometime, the battery is disconnected and a dielectric
K0 A
 C' = KC = slab of dielectric constant K is inserted between the
d
plats. How would
Q.52 Find the ratio of the potential differences that must be (i) the electric field between the plates?
applied across the parallel and the series combination (ii) the energy stored in the capacitor be affected?
of two identical capacitors, so that the energy stored in Justify your answer. [All India 2009]
the two cases becomes the same. [Foreign 2010] Sol. (i) Attempt yourself.
Sol. Let V1 and V2 are the potnetial differences across the (ii) Attempt yourself.
series and parallel combiantion of two identical
Q.55 Three identical capacitors C1. C2 and C3 of capacitances
capacitors each of capacitance C,
6F each are connected to a 12 V battery as shown
Equivalent capacitance in series combiantion
below:
C
Cs =
2
C1
Equivalent capacitance in prallel combiantion
12V C3
Cp = 2C
According to the question C2
Us = P p

1 1 Find
Cs Vs2  Cp Vp2
2 2 (i) the charge on each capacitor
(ii) the equivalent capacitances of the network
Vs2 Cp 2C (iii) the energy stored in the network of capacitors.
   [Delhi 2009C]
Vp2 Cs C
2 Sol. (i) The equivalent capacitance of C1 and C2 connected
 
in series
1 1 1
Vs2  
 4  Vs  2 C' C1 C2
Vp2 Vp
6
Vs : Vp = 2 : 1  C' = = 3F
2
 Charge, q' = C'V = (3F)12 = 36 C
Q.53 (i) Plot a graph comparing the variation of potential V
 Charge on each capacitor of C1 and C2 is 36 C
and electric field E due to a point charge Q as a function Charge on C3,
of distance R from the point charge, q3 = C3V = (6 F) × 12 = 72 = C
(ii) Find the ratio of the potential differences that must q3 = 72 C
be applied across the parallel and the series combination (ii) Equivalent capacitance of network
of two capacitors, C1 and C2 with their capacitances in C1 C2
the ratio 1 : 2, so that the energy stored in the two cases Ceq = C  C + C3
1 2
becomes the same. [Foreign 2010] 6 6
Sol. Attempt yourself. = +6
66
38
Electrostatic potential and capacitance

= 3 + 6 = 9 F
 C1 C2 
Ceq = 9 F  For series combination, C  
 C1  C2 
(iii) Energy stored in the network of capcitors
C = 5F
q '2 q '2 q2
U = U1 + U2 + U3 =    Cx = C = 5F
2C1 2C2 2C3
Cy = 4C = 4 × 5F
 C1 = C2 = C3 = 6 F = 20 F
(ii)  Charge across the combiantion
1
 U = (12 F) [q'2 + q'2 + q2] q = Ceq V
= (4 F) × 12
= 48 C
1
= [(36 C)2 + (36 C)2 + (72 C)2]  Same charge, i.e. 48 C lies on each capacitor being
(12 F)
in series combination.
U = 648  J  Same charge, i.e. 48 C lies on each capacitor being
in series combiantion.
Q.56 Two parallel plate capacitors plate X and Y have the  Potential difference across Cx,
same area of the plates and same separation between
them. X has air between the plates while Y contains a q 48 C
V1 = C = 5 C = 9.6 V
dielectric medium of r = 4 x
X Y
Potential difference across Cy,

q 48 C
V2 =   2.4 V
C y 20 C
+ –
(iii) Energy stored in capacitance X,
12V q2
(i) Calculate the capacitance of each capacitor if Ux =
2C x
equivalent of the combination is 4F.
Energy stored in capacitor Y,
(ii) Calculate the potential difference between the plates
q2
of X and Y. U = 2C
(iii) What is the ratio of electrostatic energy stored in X y

and Y? [Delhi 2009] [Since, in series, the charge will remain same]
Sol. A dielectric medium is inserted between the plates of a C y 20 F
Ux q 2 2C y
condenser in palce of air, its capacity becomes K times  U =  2 =  =4
y 2C x q Cx 5 F
of original one.
The capacitance of two capacitors are Ux : Uy = 4 : 1

0 A Q.57 A system of capacitors connected as shown in the figure


Cx = C [say]
d has a total energy of 160 mJ stored in it. Obtain the
value of the equivalent capacitance of this system and
40 A
Cy =  4C the value of Z. [All India 2009C]
d
(i) According to the problem, 10F 15F
7F
Ceq = 4 F
3F
Cx  Cy
 Ceq = C  C
x y (Z)F

C  4C 4C + –
4F = 
C  4C 5 200V

39
Electrostatic potential and capacitance

Sol. Given, total energy = 160 mJ When this charged capacitor is connected to uncharged
Let equivalent capacitance of the combiantion of capacitor,
capacitors is C.
C1, V1
According to the question,
1
CV2 = 160 × 10–3 J
2
C2, V2

1
 × C × (200)2 = 160 × 10–3 Let the common potential be V, the charge flow from
2
 Equivalent capacitance, C = 8 F first capacitor to the other capacitor unless both the
Equivalent of 7 and 3 is 10 and the 10, 10 and 15 are in capacitor attain the common potential.
series. Therefore, their equivalent capacitance  Q1 = CV1 and Q2 = CV2
Applying conservation of charge.
1 1 1 1 33 2 8
   =  Q = Q1 + Q2
C' 10 10 15 30 30
 CV = CV1 + CV2
15
C' = F ... (i) V
4  V = V1 + V2  V1 =
2
But, C' and Z are in parallel combiantion, therefore,
Total energy stored on both the capacitor.
equivalent capacitance
15 1 1
ZC [From Eq. (i)] U2 =
2
CV12 + CV22
2
4

15 1 V
2
1 V
2
+Z=8  U2 = C  + C 
4 2 2 2 2
15 32  15
 Z=8– =
4 4 2CV 2 1
U2 =  CV 2 ......(ii)
8 4
17
Z= F From Eqs, (i) and (ii), we get
4
U2 < U1
 Z = 4.25 F

Five Marks Questions It means that energy stored in the combination is less
Q.58 (i) Derive the expression for the energy stored in parallel than that stored initially in the single capacitor.
plate capacitor. Hence, obtain the expression for the
energy density of the electric field. Q.59 (i) Deduce the expression for the energy stored in a
(ii) Afully charged parallel plate capacitor is connected charged capacitor.
across an uncharged identical capacitor. Show that the (ii) Show that the effective capacitances C of a series
energy stored in the combination is less than stored combination of three capacitors C1, C2 and C3 is given
initially in the single capacitor. [Delhi 2014] by
Sol. Attempt yourself.
(ii) Initial condition C1C2 C3
C . [All India 2010C]
If we consider a charge capacitor, then its charge would C1C2  C2 C3  C3  C1
be given, q = CV
Sol. (i)Attempt yourself.
q, C, V
A B

(ii) In series combination of capacitors, same charge lie on


and energy stored in it is given by each capacitor for any value of capacitances.
1
U1  CV 2 ........(i)
2
40
Electrostatic potential and capacitance

Sol. (i) Attempt yourself.


(ii) Attempt yourself.
+Q –Q +Q –Q +Q –Q
Q.61 (a) Describe briefly the process of transferring the
charge between the two plates of a parallel plate ca-
C1 C1 C1 pacitor when connected to a battery. Derive an expres-
sion for the energy stored in a capacitor.
V1 V2 V3
(b) A parallel plate capacitor is charged by a battery to
V a potential difference V. It is disconnected from battery
(+) (–) and then connected to another uncharged capacitor of
Capactiros in series combination the same capacitance. Calculate the ratio of the energy
stored in the combination to the initial energy on the
single capacitor. [Delhi 2019]
Also, potential difference across the combination is
Sol. (a) When a parallel palate capacitor connected across
equal to the algebraic sum of potential differences
a battery, the electric currrent flows through the circuit.
across each capacitor, i.e.
As the charges reach the plate, the insulating gap does
V = V1 + V2 + V3 ......(i)
not allow the charge to move further ; ehnce positive
charges get deposited on one side of the plate and
where, V1, V2, V3 and V are the potential differences
negative charges get deposited on the other side of
across C1, C2, C3 and equivalent capacitor, respectively.
the plate. As the voltage begins to develop, the electric
q charge begins to resist further deposition of charges.
 q = C1V1  V1 =
C1 Thus, the current flowing through the circuit gradually
become less and then zero till the voltage at the capaci-
q q tor is exactly equal but oppoiste to the voltage of bat-
Similarly, V2 = , V3 
C2 C3 tery. Hence capacitor becomes charged.
 Total potential difference [from Eq. (i)]

q q q – – – – ––
V=   C –
V
C1 C2 C3 + + + +++ +

V 1 1 1 1 1 1 1
      
q C1 C 2 C 3 C C1 C 2 C 3 Energy stored in a capacitor :
Q and –Q are charges on the plates and produces a
V 1
[  where C is equivalent capacitance of
q C' 
uniform electric field E =  between the plates and a
combination] 0

1 C2 C3  C3C1  C1C2 q
or C  C1C2C3 potential difference V = .......(i)
C

C1C 2C3
 C +Q –Q
C1C 2  C2 C3  C3C1

Q.60 (i) Show that in a parallel palte capacitor, if the medium


dq
between the plates of a capacitor is filled with an
insulating substance of dielectric constant K, its
capacitance increases. If a charge dq is transported in steps from negative
(ii) Deduce the expression for the energy stored in a charged plate to positive charged plate, till charges
capacitor of capacitance C with charge Q. rises to +Q and –Q, then
[Delhi 2009C]
41
Electrostatic potential and capacitance

Work done dW = dq. V .......(ii) Q.62 A parallel plate capacitor of capacitance 'C' is charged
From equations (i) and (ii), we get to 'V' volt by a battery. After some time the battery is
disconnected and the distance between the plates is
q
dW = dq   doubled. Now a slab of dielectric constant 1 < K < 2 is
C
introduced to fill the space between the plates. How
Total electrostatic potnetial energy stored can be given will the following be affected ?
as (i) The electric field between the plates of the capaci-
tor ?
Q
Q2

q (ii) The energy stored in the capacitor
UW .dq 
C 2C Justify your answer in each case.
0
[All India 2019]
Sol. (i) The electric field between the plates is
Q2
U
2C V
E=
(b) Energy stored in a capacitro d

The distance between plates is doubled, d = 2d


1 1 1 Q2
 QV  CV 2 
2 2 2 C V'  V  1 1 E 
 E'       
d '  K  2d 2  K 
Capacitance of the (parallel) combiantion = C + C =
2C Therefore, if the distance between the plates is
Hence, total charge Q, remains the same. double, the electric field will reduce to one half.
(ii) As the capacitance of the capacitor
1 Q2
 Initial energy (single capacitor) =
2 C  0 KA 0 KA 1
C'    C
d' 2d 2
1 Q2
and final energy (combined capacitor) =
2 C Q2
Energy stored in the capacitor is U =
2C
Final energy 1
 Initial energy  2 New energy,

Q2 Q2  Q2 
U'    2  2U
2C ' 2(1/ 2)C  2C 
 

Therefore, when the distance between the plates is


doubled, the capacitance reduces to half and the en-
ergy stored in the capacitor becomes double.

42
Electrostatic potential and capacitance

EXERCISE 2
Electric potential and Potential difference Q.8 Two tiny spheres carrying charges 1.5  C and 2.5  C
Q.1 A regular hexagon of side 10 cm has a charge 5  C at are located 30 cm apart. Find the potential at the
each of its vertices, then potential at the centre of the mid point of the line joining the two charges.
hexagon is: [NCERTEXERCISE] [NCERTEXERCISE]
(1) 2.7 × 106 V (2) 2.7 × 108 V (1) 105 V (2) 2.4 × 105 V
(3) 2.7 × 1010 V (4) 2.7 × 1012 V (3) 4.4 × 106 V (4) 3 × 107 V

Q.9 A spherical conducting shell of inner radius r1 and


Q.2 Two charges 2  C and –2  C are placed at points A outer radius r2 has a charge Q.A charge q is placed at
and B 6 cm apart. Identify an equipotential surface of the centre of the shell. What is the surface charge
the system is. [NCERTEXERCISE] density on the inner and outer surfaces of the shell?
(1) The plane parallel to AB [NCERTEXERCISE]
(2) The plane having 60º angle with AB q (Q  q)
q Q  q
(3) The plane having 30º angle with AB (1) , (2) ,
(4) The plane normal to AB and passing through its 4r12 4r22 4r12 4 r22
mid point Q  q q
(3) , (4) None of the above
4r12 4 r22
Q.3 In the above question the direction of the electric field
at every point on equipotential surface will be :
[NCERTEXERCISE] Q.10 Figure below shows a charge array known as an
(1) Parallel to the direction of AB ‘electric quadrupole’. For a point on the axis of the
quadrupole, obtain the dependence of potential on r
(2) Normal to the plane in direction AB
for r/a > > 1. [NCERTEXERCISE]
(3) Any direction space a a
(4) Cannot say    
q –q –q q P
r
Q.4 A spherical conductor of radius 12 cm has a charge of (1) 1/r (2) 1/r2
1.6 × 10–7 C distributed uniformly on its surface. What (3) 1/r3 (4) r0
is the electric field inside the sphere.
[NCERTEXERCISE] Equipotential surface
(1) 105 NC–1 (2) 4.4 × 104 NC–1 Q.11 Equipotentials at a great distance from a collection of
(3) Zero (4) None of these charge whose total sum is not zero are approximately
[NCERTEXEMPLAR]
Q.5 A spherical conductor of radius 12 cm has a charge of (1) spheres (2) planes
1.6 × 10–7 C distributed uniformly on its surface. What (3) paraboloids (4) ellipsoids
is the electric field just outside the sphere.
Q.12 Figure shows some equipotential lines distributed in
[NCERTEXERCISE] space. A charged object is moved form point A to point
(1) 105 NC–1 (2) 4.4 × 104 NC–1 B. [NCERTEXEMPLAR]
(3) Zero (4) None of these
20V 40V
Q.6 A spherical conductor of radius 12 cm has a charge of
1.6 × 10–7 C distributed uniformly on its surface. What
is the electric field at a point 18 cm from the center of A B
the sphere? [NCERTEXERCISE]
(1) Zero (2) 105 N C-1
(3) 4.4 × 104 N C-1 (4) None of these
10V 20V 30V 40V 50V 10V 30V 50V
Q.7 A cube of side b has a charge q at each of its vertices.
Determine the potential and electric field due to this Fig. (i) Fig. (ii)
30V
charge array at the center of the cube.
[NCERTEXERCISE]
4q 6 3q 4q
(1) A B
30 b , 0 b2 (2) 3 b , zero
0

q 6 3q
(3)  b , (4) None of the above
0 0 b2 10V 20V 40V 50V
Fig. (iii)
43
Electrostatic potential and capacitance

(1) The work done in fig. (i) is the greatest. Relation between electronic field & Potential differnce
(2) The work done in fig. (ii) is least Q.18 In a Van de Graaff type generator a spherical metal
(3) The work done is the same in fig. (i), fig (ii) and fig. shell is to be a 15 × 106 V electrode. The dielectric
(iii) strength of the gas surrounding the electrode is
(4) The work done in fig. (iii) is greater than fig. (ii) but 5 × 107 Vm–1 . What is the minimum radius of the
equal to that in fig. (i). spherical shell required? [NCERTEXERCISE]
(1) 9 m (2) 6 m
Q.13 Equipotential surfaces [NCERTEXEMPLAR] (3) 3 m (4) None above
(1) are closer in regions of large electric fields compared
Q.19 The electrostatic potential on the surface of a charged
to regions of lower electric fields
conducting sphere is 100V. Two statements are made in
(2) will be more crowded near sharp edges of a
this regard S1 at any point inside the sphere, electric
conductor
intensity is zero S2 at any point inside the sphere, the
(3) will be more crowded near regions of large charge
electrostatic potential is 100V. Which of the following
densities
is a correct statement ? [NCERTEXEMPLAR]
(4) will always be equally spaced
(1) S1 is true but S2 is false
(2) Both S1 and S2 are false
Work done in an External field (3) S1 and S2 are true
Q.14 A charge of 8 mC is located at the origin. Calculate the (4) S1 is false but S2 is true
work done in taking a small charge of
–2 × 10 –9 C from a point p (0,0,3 cm) to a point Q.20 Consider a uniform electric field in the z –direction.
Q (0,4 cm, 0 ), via a point R ( 0,6 cm, 9 cm). The potential is constant [NCERTEXEMPLAR]
[NCERTEXERCISE] (1) in all space
(1) 1.8 J (2) 2.4 J (2) for any x for a given Z
(3) 1.2 J (4) 3.6 J (3) for any y for a given Z
(4) for the x–y plane for a given Z
Q.15 Two charges –q and +q are located at points
Q.21 The work done to move a charge along an equipotential
(0, 0, –a) and (0, 0, a ) respectively. What is the
from A to B [NCERTEXEMPLAR]
electrostatic potential at the points (0, 0, z) and
B  
(x, y, 0 ) ? [NCERTEXERCISE] (1) cannot be defined as –  E.dl
A
2P P
B  
(1) 4 (z 2  a 2 )
0
(2) 4 (z 2  a 2 )
0
(2) must be defined as –  E.dl
A

(3) is zero
P
(3) (4) None of these (4) can have a non –zero value.
(z 2  a 2 )
Q.22 In a region of constant potential [NCERT EXEMPLAR]
Q.16 Two charges –q and +q are located at points (1) the electric field is uniform
(0, 0, –a) and (0, 0, a ) respectively. Obtain the (2) the electric field is zero
dependence of potential on the distance r of a point (3) There can be no charge inside the region
from the origin when r/a>> 1. [NCERTEXERCISE] (4) the electric field shall necessarily change if a charge
(1) 1/r (2) 1/r2 is placed outside the region
3
(3) 1/r (4) None of these
Potential energy in an External Field
Q.17 Two charges –q and +q are located at points Q.23 In a hydrogen atom, the electron and proton are bound
(0, 0, –a) and (0, 0, a ) respectively. How much work is at a distance of about 0.53 Å. Estimate the potential
energy of the system in eV, taking the zero of the
done in moving a small test charge from the point (5,0,0)
potential energy at infinite separation of the electron
to (–7,0,0) along the x-axis? Does the answer change if
from proton. [NCERTEXERCISE]
the path of the test charge between the same points is
(1) 13.6 eV
not along the x-axis ? [NCERTEXERCISE]
(2) – 54.4 eV
(1) zero (2) 5 J
(3) – 27.2 eV
(3) 0.6 J (4) 15 J
(4) None of the above
44
Electrostatic potential and capacitance

Q.24 In a hydrogen atom, the electron and proton are bound Q.31 A parallel plate capacitor is to be designed with a
at a distance of about 0.53 Å.What is the minimum voltage rating 1kV, using a material of dielectric
work required to free the electron, given that its kinetic
energy in the orbit is half the magnitude of potential constant 3 and dielectric strength about 107 Vm–1
energy obtained in (a)? [NCERTEXERCISE] (Dielectric strength is maximum electric field a material
(1) 13.6 eV (2) 27.2 eV can tolerate without breakdown, i.e., without starting
(3) 54.4 eV (4) None of the above to conduct electricity through partial ionisation). For
Q.25 If one of the two electrons of a H2 molecule is removed, safety, we should like the field never to exceed, say
we get hydrogen molecular ion ( H 2 ) . In the ground 10% of the dielectric strength. What minimum area of
the plates is required to have capacitance of 50 pF.
state of a ( H 2 ) , the two protons are separated by
[NCERTEXERCISE]
roughly 1.5 Å, and the electron is roughly 1Å from 2
each proton. Determine the potential energy of the (1) 19 cm (2) 38 cm2
2
system. Specify your choice of the zero of potential (3) 9 cm (4) None of the above
energy. [NCERTEXERCISE]
(1) 13.2 eV (2) 19.2 eV
Q.32 A parallel plate capacitor is connected to a battery as
(3) –13.2 eV (4) –19.2 eV
shown in figure. Consider two situations.
Q.26 A positively charged particle is released from rest in an
uniform electric field. The electric potential energy of K
the charge [NCERTEXAMPLAR]
(1) remains a constant because the electric field is
uniform
(2) increases because the charge moves along the C
electric field
(3) decreases because the charge moves along the E
electric field
(4) decreases because the charge moves opposite to
the electric field. A. key K is kept closed and plates of capacitors are
moved apart using insulating handle
Electronic of conduction
Q.27 If a conductor has a potential V 0 and there are no B. key K is opened and plates of capacitors are moved
charges anywhere else outside, then apart using insulating handle. Choose the correct
[NCERTEXEMPLAR] option(S).
(1) there must be charges on the surface or inside itself
[NCERTEXEMPLAR]
(2) there cannot be any charge in the body of the
conductor (1) In A Q remains same but C changes
(3) there must be charges only on the surface (2) In B V remains same but C changes
(4) there must be charges inside the surface (3) In A V remains same and hence Q changes
Parallel plate capacitor (4) In B Q remains same and hence V changes
Q.28 In a parallel plate capacitor with air between the plates,
each plate has an area of 6 ×10–3 m2 and the distance Effect of Dielectoric on capacitance
between the plates is 3 mm. Capacitance of the
Q.33 A parallel plate capacitor with air between the plates
capacitor and if capacitor is connected to a 100 V
supply, The charge on each plate of the capacitor is has a capacitance of 8 pF(1pF= 10–12 F). What will be
near to : [NCERTEXERCISE] the capacitance if the distance between the plates is
(1) 9 pF, 0.9 × 10–9C (2) 18 pF, 1.8 × 10–9C reduced by half , and the space between then is filled
(3) 27 pF, 2.7 × 10–9 C (4) None of the above
with a substance to dielectric constant 6 ?
Q.29 The area of the plates of a 2 F parallel plate capacitor, [NCERTEXERCISE]
given that the separation between the plates is (1) 16 pF (2) 32 pF
0.5 cm? [NCERTEXERCISE] (3) 96 pF (4) 8 pF
(1) 750 km2 (2) 2260 km2
(3) 1130 km2 (4) None of the above
COMBINATION OF CAPACITORS
Q.30 If the force on each plate of a parallel plate capacitor Q.34 Three capacitors of capacitances 9 pF are connected
has a magnitude equal to QE, where Q is the charge on
the capacitor ,and E is the magnitude of electric field in series. What is the total capacitance of the
between the plates. Find the value of n. combination ? [NCERTEXERCISE]
[NCERTEXERCISE] (1) 3 pF (2) 27 pF
(1) 3/2 (2) 1/2 (3) 12 pF (4) None of the above
(3) 5/2 (4) 1
45
Electrostatic potential and capacitance

Q.35 Three capacitors of capacitances 9 pF are connected Q.40 A parallel plate capacitor is made of two dielectric blocks
in series. Determine the potential difference on each in series. One of the blocks has thickness d 1 and
dielectric constant K1 and the other has thickness d2
capacitor of the combination is connected to a 120 V
and dielectric constant K2 as shown in figure. This
supply? [NCERTEXERCISE] arrangement can be thought as a dielectric slab of
(1) 120 V (2) 90 V thickness d(= d1 + d2) and effective dielectric constant
(3) 40 V (4) None of the above k. the value of k is- [NCERTEXEMPLAR]

Q.36 Three capacitors of capacitances 2 pF, 3 pF and 4 pF d1 K1


are connected in parallel.What is the total capacitance
of the combination? [NCERTEXERCISE] d2 K2
(1) 90 pF (2) 9 pF
(3) 3 pF (4) 1 pF
K1d1  K 2d 2 K1d1  K 2d 2
(1) d1  d 2 (2) K1  K 2
Q.37 Three capacitors of capacitances 2 pF, 3 pF and 4 pF
are connected in parallel. Determine the charge on each K1K 2  d1  d 2  2K1K 2
capacitor if the combination is connected is connected (3)
 K1d 2  K 2d1  (4) K  K
1 2
to a 100 V supply ? [NCERTEXERCISE]
(1) 100 pF, 200 pF, 300 pF Q.41 In the circuit shown in figure initially key K1 is closed
(2) 200 pF, 300 pF, 400 pF key K2 open. Then K1 is opened and K2 is closed (order
(3) 400 pF, 500 pF, 400 pF is important). [Take Q'1 and Q '2 as charges on C1 and
(4) None of the above C2 and V1 and V2 as voltage respectively.]
[NCERTEXEMPLAR]
Q.38 An electrical technician requires a capacitance of 2  F K1 K2
in a circuit across a potential difference of 1 kV. A large
number of 1  F capacitors are available to him each of C1 C2
which can withstand a potential difference of not more
E
than 400 V. Manimum numbers of capacitors required
will be [NCERTEXERCISE]
(1) 3 (2) 6 Then
(3) 18 (4) 24 (1) charge on C1 gets redistributed such that V1 = V2
(2) charge on C1 gets redistributed such that Q'1= Q'2
Q.39 A capacitor of 4 µF is connected as shown in the circuit. (3) charge on C 1 gets redistributed such that
The internal resistance of the battery is 0.5. The C1V1+ C2V2 = C1E
(4) charge on C 1 gets redistributed such that
amount of charge on the capacitor plates will be Q'1+ Q'2 = Q
[NCERTEXEMPLAR]
Energy in capacitors
Q.42 A 12 pF capacitor is connected to a 50 V battery. How
much electrostatic energy is stored in the capacitor?
[NCERTEXERCISE]
(1) 0.5 × 10–8 J (2) 1.5 × 10–8 J
(3) 1.5 × 10–6 J (4) 3 × 10–8 J

Q.43 A 600 pF capacitor is charged by a 200 V supply. It is


then disconnected from the supply and is connected
to another uncharged 600 pF capacitor. How much
electrostatic energy is lost in the process ?
(1) 0 (2) 4 µ C
[NCERTEXERCISE]
(3) 16 µ C (4) 8 µ C (1) 6 × 10–6 J (2) 12 × 10–6 J
(3) 24 × 10–6 J (4) 18 × 10–6 J

46
Electrostatic potential and capacitance

Q.44 The plates of a parallel plate capacitor have an area of Q.47 A spherical capacitor has an inner sphere of radius
90 cm2 each and are separated by 2.5 mm. The capacitor 12cm and an sphere of radius 13 cm. The outer sphere
is charged by connecting it to a 400 V supply. is earthed and the inner sphere is given a charge of
Electrostatic energy stored by the capacitor and the 2.5  C. The space between the concentric spheres is
energy per unit volume will be filled with a liquid of dielectric constant 32. The
[NCERTEXERCISE] capacitance of the capacitor and the potential of the
–6 –3
(1) 2.55 × 10 J, 0.113 Jm inner sphere will be [NCERTEXERCISE]
(2) 10–6 J, 0.113 Jm–3 -9
(1) 5.5 × 10 F, 10 V–2

(3) 2.55 × 10–6 J, 5 Jm–3 (2) 5.5 × 10-9 F, 4.5 × 10-2 V


(4) None of the above (3) 10–9 F, 4.5 × 10-2 V
(4) None of the abvoe
Q.45 A 4  F capacitor is charged by a 200 V supply. It is
then disconnected from the supply, and is connected Q.48 A cylindrical capacitor has two co-axial cylinders of
to another uncharged 2  F capacitor. Electrostatic length 15 cm and radii 1.5 cm and 1.4 cm. The outer
energy of the first capacitor that is lost in the form of cylinder is earthed and the inner cylinder is given a
heat and electromagnetic radiation, charge of 3.5  C. Determine the capacitance of the
[NCERTEXERCISE] system and the potential; of the inner cylinder. Neglect
(1) 10–3 J (2) 2.67 × 10–2 J end effects ( i,e., bending of field lines at the ends.).
(3) 10–2 J (4) None of the above [NCERTEXERCISE]
(1) 10-12 F, 104 V
Spherical and cylindrical capacitors (2) 1.2 × 10-10 F, 2.9 × 104 V
Q.46 A spherical capacitor consists of two concentric (3) 1.2 × 10-8 F, 2.9 × 102 V
spherical conductors, held in position by suitable (4) None of the above
insulating supports.The capacitor is given by (where
r1 and r2 are the radii of outer and inner spheres
respectively). [NCERTEXERCISE]
Charge Q
+ + +
+
+ + + +

– – ––
+ + +

–– ––

–– r2 ––– r
1

+

+
+ + +
Charge Q
4 0 40
(1) r r (r  r ) (2) (r  r )
12 1 2 1 2

4 0 r1r2
(3) (r  r ) (4) None of the above
1 2

ANSWER KEY
EXERCISE 2
Q.1 (1) Q.2 (1) Q.3(2) Q.4 (3) Q.5 (1) Q.6 (2) Q.7 (2) Q.8 (2) Q.9 (1) Q.10 (3)
Q.11 (1) Q.12 (3) Q.13 (1,2,3) Q.14 (3) Q.15 (2) Q.16 (2) Q.17 (1) Q.18 (3) Q.19 (3) Q.20 (2,3,4)
Q.21 (2, 3) Q.22 (2, 3) Q.23 (3) Q.24 (1) Q.25 (4) Q.26 (4) Q.27 (3) Q.28 (2) Q.29 (3) Q.30 (2)
Q.31 (1) Q.32 (3, 4) Q.33 (3) Q.34 (1) Q.35 (3) Q.36 (2) Q.37 (2) Q.38 (3) Q.39 (4) Q.40 (3)
Q.41 (1, 4) Q.42 (2) Q.43 (1) Q.44 (1) Q.45 (2) Q.46 (3) Q.47 (2) Q.48 (2)

47
Electrostatic potential and capacitance

SOLUTION
EXERCISE 2
1. Electrostatic Potential Now, we have to find the electric field at point P, where
OP = 18 cm = 0.18 m
and Capacitance Using the formula of electric field
Q.1 (1) : ABCDEF is a regular hexagon of side 10 cm each.
At each corner the charge q = 5µC is placed. O is the 1 q 9  109  1.6  107
E    4.4  104 N / C
centre of the hexagon. 4 0  OP 2
0.18  0.18

E D
q q Q.7 (2) : Let there is a cube of side b and its centre is O.
The charge q is placed at each of the corners. Side of
O the cube = b
F C q q
q q  

q q q
A 10 cm B  q
O
The total potential due to 6 charges at the vertices will
be : q q

6  106  5
VO  9  109 
10  102  q
q
6
VO = 2.7 × 10 V Length of the main diagonal of the cube

Q.2 (1) : Equipotential surface means the surface where  b 2  b 2  b 2  3b


potential remains same at each point. Distance of centre O from each of the vertices is r.
2µC 6cm –2µC b 3
r ......(i)
A B 2
The potential is zero at each point on the line which
1 q
passes through the mid-point C of AB is an Potential at point O due to one charge is V  
equipotential surface because (V = 0) 4 0 r
Potential at point O due to all charges placed at the
Q.3 (2) : The direction of electric field at every pont on the vertices of the cube
equipotential surface is normal to the surface.
8 1 q 8q  2
V '  8V  
Q.4 (3) : According to the property of electric field, inside 4 0  r 4 0  b 3 [From Eq. (i)]
the conductor it is zero. So, the electric field inside the
4q
sphere is zero. 
3 0 b
Q.5 (1) : Radius of spherical conductor (r) = 12cm = 0.12m Due to the symmetry of charges, the electric field
and charge on conductor (q) = 1.6 × 10-7C. around the center of the cube will cancel out in pair of
For a point just outside the sphere i.e. for a point lying equal charges situated at opposite corner of the cube.
on the surface of the sphere, the charge may be
supposed to the concentrated on the centre of the Q.8 (2) : Charge at point A is q1 = 1.5 µC = 1.5 × 10-6 C
sphere by using the formula of electric field. q1 q2
C
1 q 9  109  1.6  10 7 A  B
E  2   1  105 N / C 15 cm 15 cm
4 0 r 0.12  0.12 Charge at point B is q2 = 2.5 µC = 2.5 ×10-6C
Distance between points A and B
Q.6 (2) : Radius of spherical conductor (r) = 12cm = 0.12m AB = 30 cm = 0.3 cm
and charge on conductor (q) = 1.6 × 10-7C. Let point C be the mid-point of AB.
 AC = CB = 0.15 m

48
Electrostatic potential and capacitance

Potential at point C q2a 2 q2a 2


 
VC = potential due to charge q1 + Potential due to
charge q2 
4 0 r r 2  a 2   a2 
4 0 .r.r 2 1  2 
 r 
1 q 1 q
VC   1   2 r
4 0 AC 4 0 CB If  1, a<<r
a
.5  106 2.5  10 6  q  2a 2
 9  109    According to the question, V 
1
V 3
 0.15 0.15 
4 0 r 3 r
9  109  106

0.15
1.5  2.5
Q.11 (1) : In the problem, the collection of charges, whose
9  103 total sum is not zero with regard to great distance can
  4  2.4  105 V be considered as a point charge. The equipotentials
0.15
due to point charge are spherical in shape as electric
Q.9 (1) : The charge + Q resides on the outer surface of potential due to point charge q is given by
the shell. As the charge q is placed at the centre of the kq
shell, there is charge – q induced on the inner surface V
r
and a charge +q is induced on the outer surface of the
shell. Thus, the total charge on the inner surface of This suggest that electric potentials due to point charge
the shell is – q and on the outer surface of the shell is is same for all equidistant points. The locus of these
(Q + q). equidistant points, which are at same potential, form
spherical surface.
+q
+q Q.12 (3) : The work done by electrostatic force is given by
–q
W12=q (V2–V1). Here initial and final potentials are same
O· r1 in all three cases and same charge is moved, so work
q
· done is same in all there cases.
r2 Q
Q.13 (1,2,3) : The electric field intensity E is inversely
Therefore the surface charge density on the inner proportional to the separation between equipotential
surface the shell is : surfaces. So, equipotential surfaces are closer in regions
of large electric fields.
q
1   Since, the electric field intensities is large near sharp
4 r12 edges of charged conductor and near regions of large
The surface charge density on the outer surface charge densities. Therefore, equipotential surfaces are
Qq closer at such places.
2  
4 r22
Q.14 (3) : Charge at origin O is q0 = 8 mC = 8 × 10-3 C
Charge qp at point P = – 2 ×10-9C
Q.10 (3) : Given, AC = 2a, BP = r Distance OP = rP = 3 cm = 0.03 m
A a B a C P
 R(0, 6, 9)
q 
–q –q
q  Z
r
qQ
AP = r + a and PC = r – a
The potential at P is V. P
(0, 0, 3)
V = Potential at P due to A + Potential at P due to
B + Potential at P due to C
1  q 2q q  1  1 2 1 
V    q  
4 0  AP BP CP  4 0  r  a r r  a  qQ
O  Y
qO Q
q  r  r  a   2  r  a  r  a   r  r  a   (0, 4, 0)
  
4 0  r  r  a  r  a   X

49
Electrostatic potential and capacitance

Distance OQ = rQ = 4 cm = 0.04m The formula for the potential due to an electric dipole
W = qV  WPQ = qP(VQ–Vp) at any arbitrary point is
 1 q 1 q  1 p cos 
 2  109   O   O V  2
 4 0 OQ 40 OP  
4 0 r  a 2 cos 2  
 9  109  8  10 3 9  109  8  10 3  As according to question, if r > > a, neglect a2 as
Wpq  2  109  
 0.04 0.03 
p cos 
compared to r2, then electric potential V 
 1 1  4 0 .r 2
= – 2 × 10-9 × 9 × 109 × 8 × 10-3  
 0.04 0.03 
1
or V 
r2
3
 0.01  18  8  10  0.01
 18  8  103     1.2J
 0.0012  0.0012
Q.17 (1): Here two charges placed at Z-axis at position (0, 0,
Thus the work done in bringing the charge of – a) and (0, 0,a) They form a dipole of length 2a.
–2 ×10-9C from P to Q is 1.2J

Q.15 (2) : Here two charges placed at Z-axis at position (0, 0, Z


– a) and (0, 0,a) They form a dipole of length 2a.
Let the coordinates of C is (0, 0, z) and of D is (x, y, 0).
+q · B (0, 0, a)
(0, 0, z)C ·
Z · D (x, y, 0) O
Y

+q · B (0, 0, a)
·
(0, 0, z)C· –q A
X (0, 0, –a)
· D (x, y, 0) O
Y

At both the points (5, 0, 0) and (– 7, 0, 0) potentioal is


·A zero.
–q
X (0, 0, –a)
Q.18 (3) : Given,
V = 15 × 106 V and dielectric strength = 5 × 107 V/m
At both the points (5,0,0) and (-7, 0, 0) potential is zero
As we know that the electric field is 10% of dielectric
hence work done will also be equal to zero.
strength.
Q.16 (2) : Here two charges placed at Z-axis at position (0, 0, Safer value of electric field E = 10% of dielectric
10
– a) and (0, 0,a) They form a dipole of length 2a. strength   5  107
Let the coordinates of C is (0, 0, z) and of D is (x, y, 0). 100
E = 5 × 106 V/m
V
Z  E r
V 15  106
 r   3m
+q · B (0, 0, a) E 5  106
(0, 0, z)C · The minimum radius of the spherical shell required is
· D (x, y, 0) O 3m.
Y
Q.19 (3) : We know that Inside a hollow sphere electric field
· is zero and hence potential is constant.
–q A
X (0, 0, –a)

50
Electrostatic potential and capacitance

Q.20 (2,3,4) : Q.24 (1) : Charge on electron, qe = – 1.6 × 10-19C and charge
on proton, qp = 1.6× 10-19 C
E 1
The kinetic energy    (P.E.) [As Given]
V–V 2
P
B
V 1
l Kinetic energy    27.16  13.58 eV
2
A
Total energy = KE + PE
= 13.58 – 27.16
Equipotentials = – 13.58 eV  – 13.6 eV
Thus, work done required to free the electron is
Here, the figure electric field is always remain in the 13.6 eV
direction in which othe potential decreases steepest.
Its magnitude is given by the change in the magnitude Q.25 (4):There are two protons P1 and P2 with an electron e.
of potential per unit displacement normal to the e
equipotential surface at the point.
the electric field in z–direction suggest that 1Å 1Å
equipotential surfaces are in x–y plane. Therefore the r2 r3
potential is a constant for any xfor a given z, for any y r1
for a given z and on the x–y plane for a given z. P1 1.5 Å P2

Q.21 (2, 3) : Work done in displacing a charge particle is Distance between two protons r1 = 1.5 Å = 1.5 × 10-10 m
given by W12 = q(V2–V1) and the line integral of electrical The total potential energy of the system.
field from point 1 to 2 gives potential difference 1  q P1  q P2 q P1  q e q P2  q e 
B  
U    
V2–V1= –  E.dl for equipotential surface, V2–V1 = 0 4 0  r1 r2 r3 
A

and W = 0. U = 9 × 109

1.6 1019  1.6 1019 1.6 1019    1.6 1019 


Q.22 (2, 3) : The electric field intensity E and electric potential
V are related as E = 0 and for V = constant,  
 1.5 1010 1010
dV
0
dr
1.6 1019   1.6 1019  
This imply that electric field intensity E = 0  
1010 
Q.23 (3) : Charge on electron, qe = – 1.6 × 10-19C and charge
on proton, qp = 1.6× 10-19 C 9  109  1.6  1.6  1038  1 
Potential energy of the system U 10   1  1
10 1.5 
r = 0.53 Å
  = – 30.78 × 10-19J
e p OR
= Potential energy at infinity
30.78  10 19
+ Potential energy at a distance or 0.53 Å U  19.2 eV
1.6  10 19
1 q eq p
 0  Here, we use that the potential energy at infinity is
4 0 r zero.

9  109  1.6  1019   1.6   1019 Q.26 (4) : The positively charged particle experiences
0
0.53  1010 electrostatic force along the direction electric field i.e.,
= – 43.47 × 10-19J ( 1eV = 1.6 × 10-19J) form high electrostatic potential to low electrostatic
potential. Thus, the work is done by the electric field
43.47 1019
 =–27.2 eV on the postive charge, hence electrostatic potential
1.6 1019 energy of the positive charge decreases.
51
Electrostatic potential and capacitance

Q.27 (3) : The charge resides on the outer surface of a closed Q.31 (1) : Given :
charged conductor Voltage V = 1 kV = 1000 V
Dielectric constant K = 3 and
Q.28 (2) : Given : Area of plates (A) = 6 × 10-3 m2 Dielectric strength = 107 V/m
Distance between plates (d) = 3 mm = 3 × 10-3m and Here the electric field should be 10% of the dielectric
potential = 100 V strength due to safety reasons.
Capacitance of a parallel plate capacitor E = 10% of dielectric strength
10
 0 A 8.854  1012  6  103   107  106 V / m
C  100
d 3  103
V V 1000
C = 1.77 × 10-11 F Electric field E  d   10 3 m
d E 10 6
When the capacitor is connected to a 100 V supply, Capacitance
charge on each plate of the capacitor
q = CV = 1.77 × 10-11 × 100 K 0 A Cd 50 1012  10 3
C A 
q = 1.77 × 10-9C d K 0 8.854  10 12  3
A = 1.9 × 10-3 m2
Q.29 (3) : Given capacitance C = 2 F and separation between
plates d = 0.5 cm = 0.5 × 10-2m
Q.32 (3, 4) : Case A When key K is kept closed and plates of
0A capacitors are moved apart using insulating handle,
Capacitance of a parallel plate capacitor C 
d the separation between two plates increases which in

We can wirte  K0 A 


turn decreases its capacitance  C   and hence,
 d 
Cd 2  0.5  10 2
or A   the charge stored decreases as Q = CV (potential
 0 8.854  1012
continue to be the same as capacitor is still connected
= 1.13 × 109 m2 = 1130 km2 with cell).
This area is very large, so it is not possible that the Case B When key K is opened and plates of capacitors
capacitance of a capacitor is too large as 2 F. are moved apart using insulating handle, charge stored
by disconnected charged capacitor remains conserved
Q.30 (2) : and with the decreases of capacitance, potential
difference V increases as V= Q/ C.

+Q –Q Q.33 (3) : Using the formula of capacitance of a parallel


+ –
– plate capacitor, we get
+

+
– 0 A
+ C0 
– d
+ –
+ E (where A is the area of plates and d is the distance

+ – between two plates)
+ –
1 + – 2 0 A
d 8  1012  .......(i)
d
Now, the distance between the plates is reduced to
Force of attraction on 2nd plate due to 1st plate
d
F = QE1 half, d’ = and the space between the plates is filled
2
where E1 = electric field due to 1st plate at 2nd plate
with a dielectric of dielectric constant 6. Now, let the
 new capacitance be C. In this condition area remains
E1 = 2
0 same.
K0 A
   1  1  C (where K is the dielectric constant)
d'
F = (Q)  2   = 2 (Q)    = QE
    2
52
Electrostatic potential and capacitance

C1 C1 C1 C1
0 A n
C  6 2
d
or C = 12 × 8 × 10-12 [From Eq. (i)] n
or C = 96 × 10-12 F = 96 pF C1 C1 C1 C1 m rows

Q.34 (1) :  Given :C1 = C2 = C3 = 9 pF


and voltage V = 120 V
The total capacitance in series combination n
C1 C1 C1 C1
1 1 1 1 1 1 1
       1000 V 
Cs C1 C2 C3 9 9 9
As the potential difference across each row is 1000 V.
1 3 So, the potential difference across each capacitor
  Cs  3pF
Cs 9 1000

n
Minimum number of capacitors that must be connected
Q.35 (3) :  Given :C1 = C2 = C3 = 9 pF
in series in a row are
and voltage V = 120 V
In series, the potential is distributed. 1000
  400  n = 2.5
It V' is potential across there identical capacitor then n
V Here n is the number of capacitors, so it should be a
If V = 3V'  V' = whole number. If we take n = 2, then potential difference
3
across each capacitor is 500 V. Here according to
120
 V' =  40 Volt question a capacitor can bear only 400 V, so they burst.
3
We take the value of n = 3.
Thus, the potential difference across each capacitor is So, the capacitance of each row (in series)
40V.
1 1 1 1 3 3
     C ' 
C' 1 1 1 1 1
Q.36 (2) : Given, C1 = 2 pF, C2 = 3 pF and C3 = 4 pF
The total capacitance of the parallel combination is 1 m
The total capacitance of m rows is m  
given by 3 3
Cp = C1 + C2 + C3 = 2 + 3 + 4 = 9 pF. According to question, the total capacitance required
is 2µF.
Q.37 (2) : Given, C1 = 2 pF, C2 = 3 pF and C3 = 4 pF m
So, 2  m=6
Let the charges on the capacitors C1, C2 and C3 be q1,q2 3
and q3 respectively. The potential difference across Thus, the total number of capacitor = m × n = 3 × 6 = 18
each of the capacitors is same as 100 V. So, 1 µF capacitors are connected that of 6 rows having
C1 3 capacitors in each row.
q1
C2 Q.39 (4) : Current flows through 2. resistance from left to
q2 right, is given by
C3
V 2.5V
q3 I  =1A
R  r 2  0.5

 100 V  The potential difference across 2 resistance


Charge q1 at C1 = C1V = 2 × 100 = 200 pC V = IR = 1 × 2 = 2V
Charge q1 at C2 = C2V = 3 × 100 = 300 pC Since, capacitor is in parallel with 2 resistance, so it
Charge q1 at C3 = C3V = 4 × 100 = 400 pC also has 2V potential difference across it.
The charge on capacitor
q = CV = (2µF) × 2V = 8µC
Q.38 (3) : The required capacitance C = 2µ F
Potential difference V = 1 kV = 1000 V
53
Electrostatic potential and capacitance

Q.40 (3) : The capacitance of parallel plate capacitor filled


C1C 2  V1  V2 
2
with dielectric block has thickness d 1 and dielectric
2  C1  C 2 
Loss in energy (E) =
constant K1 is given by

K10 A 600  10 12  600  10 12  200  0 


2
C1  E = 6×10-6J
d1 2  600  600   10 12
Similarly, capacitance of parallel plate capacitor filled Thus the 6 × 10-6 J amount of electrostatic energy is
with dielectric block has thickness d2 and dielectric lost in the sharing of charges.
constant K2 is given by
Q.44 (1) : Area of plates A = 90 cm2 = 90 × 10-4 m2
K  A
C2  2 0 Distance between plates d = 2.5 mm = 2.5 × 10-3m
d2
and potential difference across capacitor V = 400 V
Since, the two capacitors are in series combination ,
A
the equivalent capacitance is given by
1 1 1
 
C C1 C2 d

OR

K10 A K 2 0 A (i) Electrostatic energy U  1 CV 2


2
C1C 2 d1 d2 K1K 2 0 A
C   
C1  C2 K1 0A K 
2 0 A K1d 2  K 2d1  0 A 8.854  1012  90  10 4
d1 d2 C 
d 2.5  103
But the equivalent capacitances is given by 1
 U CV 2
K 0 A 2
C
d1  d 2 1 8.854  1012  90  104
   400  400
On comparing, we have 2 2.5  103

K K d  d   2.55  10 6 J
K 1 2 1 2 (ii) Volume of the capacitor
K1d 2  K 2d1
= A × d = 90 × 10-4 × 2.5 × 10-3 = 2.25 × 10–5 m3
Energy stored per unit volume = Energy/ Volume
Q.41 (1, 4) : The charge stored by capacitor C 1 gets
2.55  10 6
redistributed between C1 and C2 till their potentials   0.113J / m3
become same i.e., V2 = V1. By law of conservation of 2.25  105
charge, the charge stored in capacitor C1 equal to sum
of charges on capacitors C1 and C2 when K1 is opened Q.45 (2) : Given, C1 = 4µF, C2 = 2µF,
and K2 is closed i.e., V1 = 200 V and V2 = 0

1 C1C 2  V1  V2 
Q'1 + Q'2 = Q 2

Loss in energy 
Q.42 (2) : Given, voltage connected across the capacitor
2  C1  C2 
V = 50 V and Capacitance of the capacitor
1 4  2  10  200  0 
12 2
C = 12 pF , V = 50 Volt  
Energy stored in the capacitor 2  4  2   106
1 1 8
E  CV 2   12  1012  50  50  1.5  10 8 J   102
2 2 3
Loss in energy = 2.67 × 10-2 J
Q.43 (1) : Given, This loss in energy is equal to the energy dissipated
C1 = 600 pF , V1 = 200 Volt in the form of heat and electromagnetic radiation.
C2 = 600 pF, V2 = 0
54
Electrostatic potential and capacitance

Q.46 (3) :  The potential difference between two spheres Q.48 (2) : Given :
V = V2 – V1 Length of the capacitor l = 15 cm
Radius of inner cylinder a = 1.4 cm
1 Q 1 Q Q 1 1
V        Radius of outer cylinder b = 1.5 cm
4 0 r2 4 0 r1 4 0  r2 r1  Charge q = 3.5 µC = 3.5 × 10-6C
The capacitance of the cylindrical capacitor
Q  r1  r2 
V   2 0 l
4 0  r1r2  C
 b
Capacitance of the spherical capacitor log e  
 a
Q Q4 0 r1r2 4 0 r1r2
C  
V Q  r1  r2  r1  r2

= 15 cm
Q.47 (2) : a

b
r2

O
r1
2  3.14  8.85  1012  15  102

 1.5  102 
2.303log10 
 1.4  102 

(i) Capacitance of a spherical capacitor


2  3.14  8.85  1012  15  10 2
  1.2  10 10 F
4 0 Kr1r2 2.303  log1.5  log1.4 
4
1 32  12  13  10
C  
r1  r2 9  109 13  12  102
q 3.5 106
C = 5.5 × 10-9 F Potential (V)    2.9 104 V
(ii) Electric potential of inner sphere C 1.2 1010

q 2.5  106
V   4.5  102 V
C 5.5  10 9

55

You might also like